Você está na página 1de 107

CÁLCULO II

PROF.A MA. MIRIAM EULALINA MARTINS FROTA


Prof. Me. Ricardo Benedito de Oliveira
REITOR

Reitor:
Prof. Me. Ricardo Benedito de
Oliveira
Pró-reitor:
Prof. Me. Ney Stival
Gestão Educacional:
Prezado (a) Acadêmico (a), bem-vindo Prof.a Ma. Daniela Ferreira Correa
(a) à UNINGÁ – Centro Universitário Ingá.
PRODUÇÃO DE MATERIAIS
Primeiramente, deixo uma frase de
Sócrates para reflexão: “a vida sem desafios Diagramação:
não vale a pena ser vivida.” Alan Michel Bariani
Thiago Bruno Peraro
Cada um de nós tem uma grande
responsabilidade sobre as escolhas que Revisão Textual:
fazemos, e essas nos guiarão por toda a vida Gabriela de Castro Pereira
acadêmica e profissional, refletindo diretamente Letícia Toniete Izeppe Bisconcim
em nossa vida pessoal e em nossas relações Mariana Tait Romancini
com a sociedade. Hoje em dia, essa sociedade
é exigente e busca por tecnologia, informação Produção Audiovisual:
e conhecimento advindos de profissionais que Heber Acuña Berger
possuam novas habilidades para liderança e Leonardo Mateus Gusmão Lopes
sobrevivência no mercado de trabalho. Márcio Alexandre Júnior Lara

De fato, a tecnologia e a comunicação Gestão da Produção:


têm nos aproximado cada vez mais de pessoas,
Kamila Ayumi Costa Yoshimura
diminuindo distâncias, rompendo fronteiras e
nos proporcionando momentos inesquecíveis.
Fotos:
Assim, a UNINGÁ se dispõe, através do Ensino a
Shutterstock
Distância, a proporcionar um ensino de qualidade,
capaz de formar cidadãos integrantes de uma
sociedade justa, preparados para o mercado de
trabalho, como planejadores e líderes atuantes.

Que esta nova caminhada lhes traga


muita experiência, conhecimento e sucesso.

© Direitos reservados à UNINGÁ - Reprodução Proibida. - Rodovia PR 317 (Av. Morangueira), n° 6114
UNIDADE ENSINO A DISTÂNCIA

01
DISCIPLINA:
CÁLCULO II

EQUAÇÕES DIFERENCIAIS
PROF.A MA. MIRIAM EULALINA MARTINS FROTA

SUMÁRIO DA UNIDADE

INTRODUÇÃO ............................................................................................................................................................. 4
1 - EQUAÇÕES DIFERENCIAIS ORDINÁRIAS DE PRIMEIRA ORDEM ................................................................... 5
2 - EQUAÇÕES DIFERENCIAIS ORDINÁRIAS DE SEGUNDA ORDEM .................................................................. 12
3 - A TRANSFORMADA DE LAPLACE ...................................................................................................................... 14
4 - CONSIDERAÇÕES FINAIS .................................................................................................................................. 17

WWW.UNINGA.BR 3
ENSINO A DISTÂNCIA

INTRODUÇÃO
Após o estudo do cálculo diferencial e integral para funções reais de uma variável real
em que são estudados os conceitos de derivada e integral, a sequência natural é o estudo das
equações diferenciais ordinárias. Esta teoria é bem ampla e está diretamente relacionada com
importantes aplicações às Ciências Naturais e Engenharia. Nosso objetivo, então, é apresentar
uma breve introdução a esta teoria, pois um estudo completo demandaria muito espaço e tempo
o que foge ao escopo deste material.

Para leitor interessado em um aprofundamento nesta teoria, existem excelentes


referências bibliográficas, diante disso, indicamos como leitura complementar:
Equações Diferenciais Elementares e Problemas de Valores de Contorno, William
E. Bouce & Richard C. Diprima, 10a Edição – Rio de Janeiro: LTC, 2015. (indicação
de leitura).

CÁLCULO II | UNIDADE 1

WWW.UNINGA.BR 4
ENSINO A DISTÂNCIA

1 - EQUAÇÕES DIFERENCIAIS ORDINÁRIAS DE


PRIMEIRA ORDEM
De modo geral, uma Equação Diferencial Ordinária (EDO) é uma equação envolvendo
uma função incógnita (necessariamente uma função de uma variável) e suas derivadas (derivadas
ordinárias). Para a abordagem de tais equações, é conveniente considerar uma classificação
quanto à ordem e tipos, visto que a ordem de uma EDO é a ordem da maior derivada que aparece
na equação. Por exemplo, as equações:

são EDOs de primeira ordem, enquanto que

são EDOs de segunda ordem. Nesta seção vamos nos restringir às Equações Diferenciais
Ordinárias de primeira ordem (ordem 1).

CÁLCULO II | UNIDADE 1
Uma função y=φ(x) é uma solução de uma EDO, em um intervalo aberto I=(a,b), que
pode ser a=-∞ e b= ∞, se ao substituirmos y=φ(x) e suas derivadas na equação obtemos uma
identidade válida para todo xЄI.

Exercício 1: Verifique que a função y=e2x é uma solução da EDO y’- y=e2x em I=(-∞,∞).

Solução: Inicialmente, observamos que a função y=e2x é derivável em toda a reta, e sua
derivada é dada por y ’ (x)=2e2x. Portanto, substituindo na equação temos:

y’-y=2e2x-e2x=e2x para todo x Є R,

o que mostra que y=e2x é uma solução da EDO em toda reta real.

A função y=e2x não é a única solução da EDO y’-y=e2x. Na verdade, podemos ver
que para cada constante C a função y=e2x+C ex é uma solução da EDO em toda
reta, ou seja, existem infinitas soluções. De fato, derivando obtemos y’=2e2x+Cex.
Então substituindo na equação temos
y’-y=[2e2x+Cex ]-[e2x+C ex ]=e2x,para todo x ЄR.

WWW.UNINGA.BR 5
ENSINO A DISTÂNCIA

Sobre o assunto em estudo, como motivação, indicamos a vídeo aula:


https://www.youtube.com/watch?v=XHyX5M6GO6w onde é possível assistir a
uma interessante introdução ao tema.

Continuando com a classificação, dividimos as EDOs de primeira ordem em dois grupos


distintos: as EDOs lineares e as EDOs não-lineares. Precisamente, uma EDO de primeira ordem
é dita linear se puder ser expressa na seguinte forma:

y’+p(x)y=q(x). (1)

Existe um método geral para resolver qualquer EDO de primeira ordem, linear – isto é
um resultado surpreendente e muito raro de se encontrar. Vejamos como se apresenta o método:
suponhamos que, para isto, as funções p=p(x) e q=q(x) sejam funções contínuas em um intervalo
aberto I ⊂ R. Então está bem definida a função μ(x)=e ∫p(x)dx, denominada por fator integrante.
Multiplicando ambos os membros da EDO (1) pelo fator integrante μ temos:

CÁLCULO II | UNIDADE 1
ou ainda,

Integrando ambos os membros da equação em relação x obtemos:

onde C é uma constante arbitrária (constante de integração). Logo, a solução geral da


EDO (1), no intervalo I, é dada pela expressão:

Aqui, entendemos por solução geral a família de soluções a um parâmetro, isto é, uma
solução contendo uma constante arbitrária, de modo que para cada escolha da constante C temos
uma solução particular da EDO (1). O gráfico de cada solução particular é chamado de curva
integral da equação e a solução geral determina uma família de curvas integrais.
Exercício 2: Resolva a EDO y’=2y.

Solução: Inicialmente, reescrevemos a EDO na forma (1), ou seja, y’-2y = 0. É claro que
se trata de uma EDO de primeira ordem, linear, com p(x)=-2 e q(x)=0. O fator integrante é μ(x)
= e∫-2dx = e-2x. Multiplicando a EDO pelo fator integrante resulta:

WWW.UNINGA.BR 6
ENSINO A DISTÂNCIA

Integrando em relação a x,temos para x ЄR. Isto é a solução


geral da EDO em toda a reta.

A EDO do exercício 2 é um modelo que afirma que a taxa de variação de y é proporcional


a quantidade y, sendo a constante de proporcionalidade igual a dois. Este é um caso particular de
um modelo de crescimento exponencial que consideraremos logo mais a frente.
Para muitos problemas práticos impõe-se, além da EDO, uma condição adicional que
determina um valor específico para a constante arbitrária que aparece na solução geral. Esta
condição adicional consiste em especificar um valor determinado y0 para a solução y, em um
ponto x0 do intervalo I, isto é, y(x0) = y0, chamada de condição inicial. A EDO e mais uma condição
inicial formam o que chamamos de problema de valor inicial (PVI). Assim, um PVI para uma
EDO de primeira ordem, linear, é um problema do tipo:

Se p e q são contínuas no intervalo I e x0∈ I, então para cada y0 ∈ R existe uma única
solução para o (PVI).

Exercício 3: Resolva o PVI:

CÁLCULO II | UNIDADE 1
Solução: A EDO é de primeira ordem e linear. O fator integrante é
Multiplicando a EDO pelo fator μ, temos:

Integrando, resulta:

Integrando por partes, duas vezes, obtemos que


Substituindo na expressão anterior, encontramos a solução geral da EDO:

Para determinarmos o valor da constante C, usamos a condição inicial, isto é:



Portanto, a solução do PVI é a função:

As EDOs de primeira ordem, e lineares, são importantes para as aplicações, especialmente


para aquelas grandezas que crescem ou decrescem a uma taxa que é proporcional à quantidade
presente em cada instante. Estes modelos são ditos modelo de crescimento ou de decrescimento
exponencial. Em termos precisos, uma grandeza y = y (t), que depende do tempo t, tem um
modelo de crescimento exponencial se a solução da EDO é de primeira ordem e linear:

WWW.UNINGA.BR 7
ENSINO A DISTÂNCIA

onde a constante de proporcionalidade k>0 é chamada de constante de crescimento


exponencial. Analogamente, afirmamos que uma grandeza tem um modelo de decrescimento
exponencial, se ela é solução da EDO de primeira ordem e linear:

e neste caso, a constante de proporcionalidade é chamada de constante de


decrescimento exponencial. Repetindo o mesmo raciocínio aplicado no exercício 2, encontramos
que a solução geral de (4), que é dada por . É fácil ver que a constante
Logo, tem um modelo de crescimento exponencial, com constante de crescimento
e satisfaz se, e somente se,

Analogamente, tem um modelo de decrescimento exponencial, com constante


de decrescimento e satisfaz se, e somente se,

Exercício 4: Em condições ideais, podemos considerar que a população de uma cultura

CÁLCULO II | UNIDADE 1
de bactérias tem um modelo de crescimento exponencial. Sabendo que cada célula da bactéria
E. Colli divide-se em duas a cada 20 minutos, se iniciamos uma cultura com uma única célula e
for o número de células presentes na cultura após minutos, determine:
a) o PVI, cuja solução seja a função e uma fórmula para esta solução

b) o número de células da bactéria presentes na cultura, após 2 horas;

c) o tempo necessário para que a cultura atinja 1 milhão de células.

Solução:
a) Conforme definimos, se tem um modelo de crescimento exponencial, então,
a taxa de variação de é proporcional à quantidade . Como iniciamos a cultura com uma única
célula, a condição inicial imposta é Então, o PVI tem a solução a partir da função

é e Para determinarmos a constante de crescimento usamos

a informação de que cada célula da bactéria E. Colli divide-se em duas a cada 20 minutos. Assim,

devemos ter que


Portanto,


b) O número de células da bactéria, presentes na cultura, após 2 horas (120 minutos) é

WWW.UNINGA.BR 8
ENSINO A DISTÂNCIA

c) Para encontrar o tempo, é necessário para que a cultura atinja 1 milhão ( de


células devemos resolver a equação Então,

Exercício 5: (Lei de resfriamento de Newton) A lei de resfriamento de Newton expõe


que a taxa de resfriamento de um objeto é proporcional à diferença das temperaturas entre o
objeto e o meio ambiente. Durante a investigação de um assassinato, a polícia técnica, ao chegar
ao local do crime, coletou as seguintes informações: às 13:30 a temperatura do corpo estava a
e uma hora depois, a temperatura caiu para ; além disso, a temperatura na sala onde
se encontrava o corpo era constante de . Admitindo-se que a temperatura do corpo, no
momento do assassinato, era – a normal – de , a que horas aconteceu o assassinato?

Solução: Se é a temperatura do corpo (medida em graus célsius) no instante


de tempo (medido em minutos) e é a temperatura do ambiente, que supomos
ser constante, matematicamente, a lei de resfriamento de Newton é modelada pela EDO:
onde é a constante de resfriamento (constante de proporcionalidade).
Fazendo a mudança de variável , temos que e

CÁLCULO II | UNIDADE 1
Ou seja, a

nova variável deve satisfazer ao PVI: . Então devemos ter e,


consequentemente, . Para encontrarmos o valor da constante consideramos
que às 13:30 estamos em um instante de tempo , assim, após uma hora, estaremos no instante
de tempo . Pelos dados coletados, temos que

Portanto,

ou seja,

Logo,

Agora, vamos determinar o valor de tal que Para isto:

Portanto, do momento do assassinato ( , até a temperatura (do corpo) atingir


quando era 13:30, passaram-se 116,1 minutos (1 hora e 56 minutos e 6 segundos). Portanto, o
assassinado aconteceu aproximadamente às 11:33:54.

WWW.UNINGA.BR 9
ENSINO A DISTÂNCIA

Diferentemente, das EDOs de primeira ordem lineares, as não-lineares não podem ser
tratadas por um método geral. Entretanto, ainda é possível de se desenvolver métodos de soluções
para algumas situações (tipos) muito especiais.

Existem, também, outros tipos de EDO de primeira ordem, interessantes para


aplicações, que possuem método de resolução, como por exemplo, as EDOs
Homogêneas e as EDOs Exatas; veja [6] e [7].

Uma EDO de primeira ordem é de variáveis separáveis, que pode ser expressa na forma:

onde é uma função somente da variável e é uma função somente

CÁLCULO II | UNIDADE 1
da variável y. Podemos reescrever a EDO (6) na forma:

Denotando vemos que a EDO (6) pode ser reescrita na forma

dita forma diferencial. Observe que na forma diferencial (7), no primeiro membro da
EDO, só aparece a variável e, no segundo membro, somente a variável Esta é a razão da
nomenclatura “variáveis separáveis”.

Exercício 6: Verifique que a EDO dada é uma EDO de primeira ordem de variáveis
separáveis, escreva a EDO na forma normal (6) e também na forma diferencial (7):

Solução: a) A EDO é de primeira ordem. Além disso, podemos ver que

, que é a forma normal com e

Na forma diferencial, temos a equação:

WWW.UNINGA.BR 10
ENSINO A DISTÂNCIA

b) A EDO é de primeira ordem. Além disso, podemos ver que

que é a forma normal com e

Na forma diferencial temos a equação:

c) A EDO é de primeira ordem. Além disso, podemos ver que

que é a

forma normal com e Na forma diferencial, temos a equação:

A solução geral de uma EDO de primeira ordem, de variáveis separáveis, pode ser obtida
por integração. Basta integrar a equação (7), ambos os membros:

Exercício 7: Resolva as EDOs:

CÁLCULO II | UNIDADE 1
Solução: a) Integrando o
primeiro membro em relação a e o segundo em relação a , obtemos:

b) Inicialmente, observamos que a função nula é uma solução da equação. Supondo,


então, podemos dividir ambos os membros por :

Esta última
equação define, implicitamente, como função de Podemos, neste caso, explicitar :

Logo, considerando uma constante arbitrária


podemos expressar todas as soluções numa única fórmula:

c) Novamente, vemos, de início, que a função é uma solução da EDO. Para buscar
soluções não-identificamente nulas, supondo , tem,os:

WWW.UNINGA.BR 11
ENSINO A DISTÂNCIA

Integrando a equação acima temos:

Denotando por uma constante arbitrária


a solução geral pode ser expressa por:

2 - EQUAÇÕES DIFERENCIAIS ORDINÁRIAS DE


SEGUNDA ORDEM

De modo geral, uma EDO de segunda ordem é uma equação do tipo:

onde é uma função dada. Como feito anteriormente, aqui também as EDOs de segunda
ordem se dividem em dois grupos – as equações lineares e as equações não-lineares. A EDO (8)
é linear e pode ser escrita na forma:

CÁLCULO II | UNIDADE 1
onde, e são funções dadas. Em particular, quando a função do segundo membro
é identicamente nula (isto é: ), a EDO é dita homogênea. Além disso, se as funções
e são constantes, temos uma EDO de segunda ordem, linear, homogênea, com coeficientes
constantes, ou seja, uma equação do tipo:

As EDOs de segunda ordem, lineares, homogêneas, com coeficientes constantes aparecem


no estudo de vários problemas práticos, como no estudo de vibrações mecânicas (sistemas massa-
mola) e, também, no estudo de circuitos elétricos (para aplicações consulte [??] e [??]). Portanto,
no que segue, nosso objetivo será apresentar um método para resolvê-las.
A teoria está baseada no tipo das raízes de uma equação algébrica associada, chamada
equação característica da EDO (10). Concretamente, denomina-se equação característica
associada à equação (10) – a equação quadrática:

A depender do valor do seu discriminante existem três possibilidades:


a equação característica (11) pode ter duas raízes reais distintas e , duas raízes reais iguais
(uma raiz com multiplicidade dois), ou, ainda, duas raízes complexas conjugadas
e .
Para cada uma dessas três possibilidades, temos uma respectiva forma para a solução
geral da EDO (10):

WWW.UNINGA.BR 12
ENSINO A DISTÂNCIA

1) No primeiro caso, a solução geral é ;

2) No segundo caso, a solução geral é ;

3) No terceiro caso, a solução geral é ;


onde e são constantes arbitrárias.

Exercício 8: Resolva as EDOs:

Solução: a) A equação característica associada à EDO é


que possui duas raízes reais distintas e Logo, a solução geral é dada por:

b) Neste item, a equação característica é dada por Resolvendo esta


equação, encontramos que o discriminante Portanto, temos raízes

CÁLCULO II | UNIDADE 1
reais iguais (ou uma única raiz com multiplicidade dois) Assim, a solução geral é:


c) Por fim, neste item, a equação característica associada a EDO é
cujo discriminante é
Logo, temos duas raízes complexas conjugadas
e . Então, a solução geral é:


Quando impomos duas condições iniciais, temos um problema de valores iniciais e, neste
caso, as constantes arbitrárias e são determinadas.

Exercício 9: Resolva o problema de valores iniciais

Solução: Pelo item (c) do exercício anterio,r já sabemos que a solução geral da EDO
é dada por Vamos determinar os valores das constantes
e tais que as duas condições iniciais sejam satisfeitas. Sabendo que devemos
ter que Então,
. Derivando , calculando o resultado no ponto e usando
que obtemos:

WWW.UNINGA.BR 13
ENSINO A DISTÂNCIA

Portanto, a única solução do problema de valores iniciais é:


.

3 - A TRANSFORMADA DE LAPLACE
É uma transformada integral bastante útil para resolução de equações diferenciais
ordinárias, isto porque transforma uma EDO em uma equação algébrica. O método consiste,
basicamente, em aplicar a transformada de Laplace na EDO, transformando-a em uma equação
algébrica, em seguida, resolvemos a equação algébrica e aplicamos a transformada inversa para
obter a solução da EDO.

Seja uma função seccionalmente contínua para todo tal que existem
constantes e , tais que Então, para todo
número real existe a integral (imprópria) . Em outras
palavras, está bem definida uma função , dada por

CÁLCULO II | UNIDADE 1
chamada a Transformada de Laplace de

Observação: Por simplicidade, é comum denotar a função original por uma letra
minúscula e sua correspondente, Transformada de Laplace pela mesma letra maiúscula. Ou seja,
denota a Transformada de Laplace de e denota a Transformada
de Laplace de e etc.

Exercício 10: Determine a Transformada de Laplace da função: a) função constante


b) onde é uma constante.

Solução: a) Pela definição da Transformada de Laplace, temos

Como a Transformada de Laplace é dada por uma integral, é natural esperarmos que a
linearidade do operador integral seja transferida para a Transformada. De fato, para quaisquer
funções e cujas Transformadas de Laplace existam e quaisquer constantes e temos:

WWW.UNINGA.BR 14
ENSINO A DISTÂNCIA

Usando a linearidade e a definição da Transformada de Laplace, é possível obter a


Transformada de Laplace de várias funções elementares importantes para as aplicações. Na
tabela, resumimos a Transformada de Laplace de algumas dessas funções.

CÁLCULO II | UNIDADE 1
Comentamos, no início desta seção, sobre a utilidade da Transformada de Laplace como
uma ferramenta para resolução de EDOs.

A linearidade da Transformada de Laplace é essencial para que a técnica funcione,


mas não é somente isto que basta. É, ainda, necessário propriedades sobre a
Transformada de Laplace de derivadas e integrais.

Nesta linha de raciocínio, a propriedade que permite transformar equações diferenciais em


equações algébricas é que Transformada de Laplace destrói derivadas, ou seja, a Transformada de
Laplace da derivada de uma função resulta em simples multiplicação por da sua transformada.
Para simplificar os cálculos, admita que seja continuamente diferenciável para e
exista sua Transformada de Laplace (isto é: existem constantes e , tais que
), então, usando integração por partes temos:

WWW.UNINGA.BR 15
ENSINO A DISTÂNCIA

Aqui, usamos que Logo, verificamos que:

Admitindo-se suficiente regularidade e existência das Transformadas de Laplace para f e


suas derivadas, aplicamos sucessivamente o raciocínio acima e concluímos que:

e, de modo geral, para a n-ésima derivada temos:

CÁLCULO II | UNIDADE 1
Exercício 11: Use a transformada de Laplace para resolver o problema de valores iniciais:

Solução: Aplicando a Transformada de Laplace na EDO e usando as propriedades (13) e


(15) obtemos:

Denotando, L{y(t)}=Y e usando as condições iniciais temos a equação algébrica

cuja solução é dada por

Usando frações parciais, encontramos

WWW.UNINGA.BR 16
ENSINO A DISTÂNCIA

Então,

Aplicando a inversa da Transformada de Laplace e usando a tabela, encontramos a


solução do problema

4 - CONSIDERAÇÕES FINAIS
Neste módulo, nosso objetivo principal foi apresentar uma introdução ao estudo das
equações diferenciais ordinárias, de primeira e segunda ordens, e também algumas noções sobre
a Transformada de Laplace. Vimos a estreita relação deste estudo com as aplicações à Física e
Engenharias. O desenvolvimento das ideias seguiu uma abordagem baseada na classificação das

CÁLCULO II | UNIDADE 1
equações diferenciais segundo a ordem e tipo, sendo este, dividido em dois grupos as equações
lineares e as equações não lineares. As equações de primeira ordem, lineares, apresentaram
um comportamento bastante interessante e graças a isto foi possível estabelecer uma fórmula
geral para sua solução. Quanto às equações diferenciais ordinárias de primeira ordem, não-
lineares, estudamos as chamadas equações de variáveis separáveis. Ao resolvermos uma equação
diferencial ordinária de primeira ordem encontramos uma família de funções, a um parâmetro,
translações por constantes, chamada de solução geral. Se por outro lado, adicionalmente fixamos
uma condição inicial, então uma, e somente uma, função da família (que representa a solução
geral) será a solução do problema de valor inicial, formado pela equação diferencial mais a
condição inicial.
O método para resolução das equações de segunda ordem, lineares, com coeficientes
constantes, baseou-se no tipo de raízes de uma equação algébrica, do segundo grau, associada à
equação diferencial, chamada equação característica. O método é bem simples e as aplicações
envolvem vibrações mecânicas (sistema massa mola) e também o estudo de circuitos elétricos,
dentre outras. Finalizamos o módulo com a Transformada de Laplace, que permitiu transformar
as equações diferencias ordinárias em equações algébricas. Resolvida a equação algébrica, por
meio de um processo inverso da Transformada de Laplace, chegamos à solução da equação
diferencial. Esta técnica é especialmente útil para tratar equações diferencias com a presença de
forças descontínuas.

WWW.UNINGA.BR 17
UNIDADE ENSINO A DISTÂNCIA

02
DISCIPLINA:
CÁLCULO II

CÁLCULO DIFERENCIAL
PROF.A MA. MIRIAM EULALINA MARTINS FROTA

SUMÁRIO DA UNIDADE

1 - FUNÇÕES REAIS DE VÁRIAS VARIÁVEIS REAIS ............................................................................................. 20


2 - LIMITES E CONTINUIDADE ............................................................................................................................... 31
3 - DERIVADAS PARCIAIS ....................................................................................................................................... 35
4 - DIFERENCIABILIDADE E A REGRA DA CADEIA .............................................................................................. 40
5 - DERIVADAS DIRECIONAIS, MÁXIMOS E MÍNIMOS ....................................................................................... 42
6 - CONSIDERAÇÕES FINAIS ................................................................................................................................. 48

WWW.UNINGA.BR 18
ENSINO A DISTÂNCIA

INTRODUÇÃO
Na disciplina Cálculo Diferencial e Integral I estudamos, dentre outras coisas, dois
conceitos centrais da matemática extremamente importantes para as aplicações, chamados
Derivada e Integral. Vimos que estas operações são realizadas sobre funções que modelam as
relações de dependência entre duas grandezas numéricas (escalares), o que chamamos de funções
reais de uma variável real. Resumindo, funções reais de uma variável real são os objetos centrais
do cálculo diferencial e integral I. Por outro lodo, é fácil ver que tais funções não são suficientes
para descrever todos os fenômenos e, de fato, muitas grandezas escalares dependem de duas
ou mais variáveis. Portanto, é necessário desenvolver um cálculo para estas funções e este é o
objetivo principal desta disciplina. Neste módulo, introduzimos o conceito de função real de
duas ou mais variáveis reais e estudamos limites, continuidade e derivadas para estas funções. Por
simplicidade na apresentação, quase sempre apresentamos os conceitos, inicialmente, para o caso
de funções escalares de duas variáveis (definidas numa região do plano) e depois generalizamos
para funções de 3 ou mais variáveis.

CÁLCULO II | UNIDADE 2
Há, ainda, uma generalização do Cálculo Diferencial e Integral para funções
vetoriais, isto é, funções do tipo r ⃗(t)=f(t) i ⃗+g(t) j ⃗+h(t) k ⃗. Consulte, por exemplo,
as referências [1, 2, 3, 4 e 5].

WWW.UNINGA.BR 19
ENSINO A DISTÂNCIA

1 - FUNÇÕES REAIS DE VÁRIAS VARIÁVEIS REAIS


Já observamos que as grandezas dependem umas das outras e em diversas situações,
encontramos que o valor de determinada grandeza depende de outras duas, ou mais, chamadas
variáveis independentes (livres). Vamos, inicialmente, tratar das funções reais de duas variáveis
reais e os outros casos, quando a dependência for de três, quatro ou mais variáveis, a situação
é bastante semelhante. Por exemplo, a área de um retângulo depende dos valores de seu
comprimento e sua largura , ou seja, a área é uma função real, de duas variáveis reais, dada
por . Da mesma forma, o volume de um cilindro circular reto cujo raio da
base é e a altura é é ; ou seja, o volume do cilindro é função do raio e da
altura. Neste caso, o volume é a variável dependente enquanto que o raio e a altura são variáveis
independentes. Formalmente temos a definição.

Definição: Uma função real de duas variáveis reais é uma regra (uma lei) que para cada
ponto de uma região do plano , que eventualmente pode ser todo ou parte
dele, associa um único número real .
Genericamente, denotamos uma função real de duas variáveis reais por:

CÁLCULO II | UNIDADE 2

ou, simplesmente, por para todo A região é o domínio da


função denotado por e como usual quando não se faz menção sobre o domínio da
função devemos considerar como domínio a “maior” região para o qual faz sentido a regra (ou
fórmula) que a define. O conjunto de todos os valores assumidos pela função é o conjunto

O termo curva de nível é muito usado na agrimensura e indica as curvas do terreno


que estão à mesma altura, ao se caminhar pela curva de nível a altura não muda.

WWW.UNINGA.BR 20
ENSINO A DISTÂNCIA

Exercício 1: Determine o domínio, imagem, curvas de níveis (mapa de contorno) e o


gráfico da função

Solução: É fácil ver que existe para todo ponto isto é,


Também podemos ver que dado qualquer número real é possível encontrar um par tal
que ou seja, Agora, dado um número real a curva de nível é
dada pela equação:

Portanto, as curvas de nível são retas paralelas. Vejamos para alguns valores de

O esboço do mapa de contorno

CÁLCULO II | UNIDADE 2

Figura 1 - Esboço do mapa de contorno para as curvas de nível


e , da direita para a esquerda,
respectivamente. Fonte: o autor.

Os traços do gráfico de isto é, a interseção do gráfico de com os planos coordenados


também são retas, ou seja:

WWW.UNINGA.BR 21
ENSINO A DISTÂNCIA

Já está fácil concluir que o gráfico de f é um plano, vejamos também que:

z=5-4x-3y⇒4x+3y+z-5=0

é a equação de um plano. Na figura, fazemos o esboço do gráfico de f:

CÁLCULO II | UNIDADE 2
Figura 2 - Esboço do gráfico da função f. Fonte: o autor.

Observação: A função do exemplo é um caso particular de uma função afim, de duas


variáveis, isto é, uma função do tipo:

onde a,b e c são constantes, com a e b não simultaneamente nulos. Neste caso, o domínio
será todo o plano, a imagem toda a reta real, as curvas de nível e os traços serão retas, e o gráfico
um plano.

Exercício 2: Descreva o campo escalar que determina a distância de cada ponto do


plano a um ponto fixado.

Solução: Inicialmente, consideramos um sistema de coordenadas cartesianas para


o plano, de sorte que, segundo este sistema, o ponto fixado tenha coordenadas
Então, dado qualquer ponto temos que a distância de a é:

WWW.UNINGA.BR 22
ENSINO A DISTÂNCIA

Figura 3 - Ilustração geométrica da distância de a . Fonte: o autor.

Então, o campo escalar desejado é a função real definida em todo o plano por

CÁLCULO II | UNIDADE 2
A imagem de é o intervalo A curva de nível 2 é o conjunto dos pontos
que satisfazem à equação:

isto é, o círculo de centro no ponto e raio igual a 2. De modo geral, dado


a curva de nível ou seja, não existem curvas de nível negativo. Agora,
, a curva de nível zero é somente o ponto Por outro lado,
se a curva de nível é o círculo de centro no ponto e raio igual a isto é:
Quando desenhamos algumas curvas de nível temos o que
chamamos de “mapa de contorno”:

WWW.UNINGA.BR 23
ENSINO A DISTÂNCIA

Figura 4 - Ilustração do mapa de contorno para as curvas de nível Fonte: o


autor.

O gráfico da função é uma superfície no espaço, neste caso, o cone com vértice no
ponto Para visualizar o gráfico vejamos alguns de seus traços (interseções com certos

CÁLCULO II | UNIDADE 2
planos):

Figura 5 - Traços de f. Fonte: o autor.

WWW.UNINGA.BR 24
ENSINO A DISTÂNCIA

O gráfico de f:

Figura 6 - Gráfico de Fonte: o autor.

Em particular, quando é a origem, ou seja, temos que a função toma


a forma maios simples:

Exercício 3: Encontre o domínio da função .

CÁLCULO II | UNIDADE 2
Solução: Desde que a expressão que define a função é uma razão, não podemos ter zero
no denominador. Também observamos que o numerador envolve uma raiz quadrada que só
existe para valores não negativos. Então, o domínio de deve ser a interseção dos dois conjuntos:

WWW.UNINGA.BR 25
ENSINO A DISTÂNCIA

Figura 7 - Esboço do domínio de . Fonte: o autor.

Exercício 4: Estude a função

CÁLCULO II | UNIDADE 2
Solução: Claramente, vemos que e Para as curvas de nível,
temos:

Figura 8 - Mapa de contorno de . Fonte: o autor.

WWW.UNINGA.BR 26
ENSINO A DISTÂNCIA

Os traços podem ser obtidos por:

Figura 9 - Traços z=x2 e z=y2. Fonte: o autor.

Finalmente, o gráfico de f é o paraboloide:

CÁLCULO II | UNIDADE 2
Figura 10 - Gráfico de . Fonte: o autor.

Exercício 5: Determine o domínio, a imagem, e esboce um mapa de contorno e o gráfico


da função

Solução: Como os valores de são calculados por meio de uma raiz quadrada devemos
ter:

ou seja, o domínio é o disco de centro na origem e raio igual a Também vemos que
Portanto, para todo
ou seja, Os Traços do gráfico de nos dão:

WWW.UNINGA.BR 27
ENSINO A DISTÂNCIA

Também vemos que que é a equação da esfera


de centro na origem e raio igual a dois. Como o gráfico é a parte superior da
esfera (hemisfério norte).

CÁLCULO II | UNIDADE 2
Figura 11 - Gráfico de . Fonte: o autor.

Agora, com pequenas adaptações podemos expor o que são as funções de três variáveis.
Um problema físico, onde devemos ter uma função real de três variáveis reais, é o estudo da
temperatura em um ponto de uma chapa metálica retangular, muito fina. Podemos idealizar
a chapa como um retângulo no plano e, ao aplicarmos uma fonte de calor no lado ,
este deverá viajar sobre a chapa metálica e a temperatura num ponto que varia com
o tempo Logo, a temperatura (campo escalar) em um ponto da placa é uma função real de três
variáveis reais: as coordenadas e do ponto na placa, e o tempo . Isto é

Definição: Uma função real de três variáveis reais é uma regra (uma lei) que para cada
ponto de uma região do espaço , que eventualmente pode ser todo ou
parte dele, associa um único número real .

Genericamente, denotamos uma função real de três variáveis reais por:

WWW.UNINGA.BR 28
ENSINO A DISTÂNCIA

ou simplesmente por para todo A região é o domínio da


função denotado por O conjunto de todos os valores assumidos pela função é o conjunto
imagem, isto é, O gráfico da função é o subconjunto do
espaço (que não pode ser esboçado como uma figura) dado por:

As curvas de níveis, agora, serão superfícies de nível dadas por:

Exercício 6: Determine o domínio, imagem, e faça um mapa de contorno com algumas


superfícies de nível para a função

Solução: O domínio é o conjunto ou


ainda, os pontos do espaço que satisfazem à desigualdade portanto, a bola
fechada de centro na origem e raio igual a 1 (a esfera unida com seu interior).

CÁLCULO II | UNIDADE 2

Figura 12 - Domínio de . Fonte: o autor.

Para determinar a imagem observamos que a função sendo dada por uma raiz quadrada
os valores Além disso,

WWW.UNINGA.BR 29
ENSINO A DISTÂNCIA

então, a imagem é A superfície de nível para é dada pela


equação:

ou seja, esferas centradas na origem e raio . Para o nível temos:


isto é, um único ponto Não
existem (ou são vazias) curvas de níveis ou

CÁLCULO II | UNIDADE 2
Figura 13 - Superfícies de nível k, para 0≤k<1. Fonte: o autor.

Observação: De modo similar, podemos definir uma função real de variáveis reais
como sendo uma regra (uma lei), que para cada ponto de uma região
do espaço euclidiano , que eventualmente pode ser todo ou parte dele, associa um único
número real .

Leitura Complementar: Como leitura complementar para o estudo das funções


definidas no espaço euclidiano ℝn, sugerimos: Análise no Espaço ℝn, Elon Lages
Lima, Coleção Matemática Universitária, Impa – Rio de Janeiro, 2002.

WWW.UNINGA.BR 30
ENSINO A DISTÂNCIA

2 - LIMITES E CONTINUIDADE

Do ponto de vista teórico, os conceitos de limite e continuidade, que apresentaremos


aqui, não têm diferença alguma daqueles estudados no Cálculo I para as funções de uma variável.
No entanto, surgem novidades! Isto se deve pelo fato de que quando estamos em uma dimensão,
na reta, dado um ponto só existem duas maneiras de nos aproximarmos de por valores
maiores do que (à direita de ) ou por valores menores do que (à esquerda de ). Por outro
lado, em duas dimensões, dado um ponto no plano, existem infinitas maneiras de nos
aproximarmos deste ponto.
A grosso modo, o conceito de limite de uma função surge quando desejamos investigar
o comportamento da função quando nos aproximamos de um determinado ponto. Então, dada
uma função e um ponto , de modo que seja possível nos aproximarmos deste
ponto por pontos do domínio da função, quando os valores se tornam (e permanecem)
arbitrariamente próximos de um número à medida que se torna, arbitrariamente, próximo
de afirmamos que é o limite da função quando tende a e escrevemos:

CÁLCULO II | UNIDADE 2
Usando as métricas euclidianas do plano e da reta, ou seja, usando as expressões que
calculam a distância entre dois pontos, podemos ser mais precisos e reescrever a definição de
limite:

Conforme informamos anteriormente, dado um ponto no plano, existem infinitas


maneiras de nos aproximarmos deste ponto, e, desta forma, temos que
se, e somente se, sempre que ao longo de qualquer caminho
Ou seja, não importa como nos aproximamos do ponto os respectivos valores se
aproximam de

WWW.UNINGA.BR 31
ENSINO A DISTÂNCIA

Figura 14 - Ilustrações de algumas aproximações do ponto (a,b) e os valores de f(x,y) se aproximando de


L, respectivamente. Fonte: o autor.

O resultado exposto é conveniente para provar a não existência de certos limites. Com
efeito, se encontrarmos dois caminhos distintos C1 e C2, de modo que:

CÁLCULO II | UNIDADE 2
com L1≠L2, então não existe o limite

Exercício 7: Mostre que não existem os limites

Solução:
a) Vejamos, inicialmente, o que ocorre quando nos aproximamos do ponto pelo
eixo ou seja, consideremos e façamos ao longo do

caminho Denotando , temos que para todo Portanto,

WWW.UNINGA.BR 32
ENSINO A DISTÂNCIA

Por outro lado, denotamos e façamos ao longo do


caminho Observamos que para todo Portanto,

Logo, não existe o limite:

b) Denotando

temos que:

CÁLCULO II | UNIDADE 2
de onde concluímos que não existe o limite:

c) Seja

Então, podemos ver que:

WWW.UNINGA.BR 33
ENSINO A DISTÂNCIA

o que mostra que não existe o limite:

Exercício 8: Verifique que

Solução: Inicialmente, vejamos que é limitada. De fato,

Portanto,

CÁLCULO II | UNIDADE 2
o que prova a limitação da função φ . Agora, notamos que

logo,

O conceito de limite de uma função de três ou mais variáveis é uma extensão natural do
que apresentamos para as funções de duas variáveis. Uma função real de n variáveis reais

WWW.UNINGA.BR 34
ENSINO A DISTÂNCIA

3 - DERIVADAS PARCIAIS
Quando pensamos em generalizar a ideia de derivada de uma função de uma variável,
para funções de duas (ou mais) variáveis, surge, naturalmente, o conceito de derivada parcial.
Trata-se da taxa de variação de uma função em um ponto em relação a uma de suas variáveis
independentes. Quando estudamos a derivada de uma função de uma variável, em um
determinado ponto a, exigimos que a função estivesse definida em uma vizinhança deste ponto a,
ou seja, era necessário que um intervalo aberto do tipo (a-δ,a+δ) ⊂ D(f). Isto, em outras palavras,
significa afirmar que a é o ponto interior do domínio da função. Agora, em dimensões maiores
do que um, devemos agir da mesma forma, e, para isto, introduzimos os conceitos topológicos:

a) A distância (euclidiana) entre dois pontos quaisquer e


do espaço -dimensional é dada por:

b) Dados um ponto e a bola aberta de centro em e raio


é o conjunto

c) Dados um ponto e a bola fechada de centro em e raio

CÁLCULO II | UNIDADE 2
é o conjunto

d) Dados um ponto e a esfera de centro em e raio éo


conjunto

Figura 15 - Ilustrações da bola aberta, bola fechada e esfera, respectivamente. Fonte: o autor.

É claro que . Quando a bola aberta é o


intervalo aberto a bola fechada é o intervalo fechado e a esfera
consiste somente por dois pontos Para a bola aberta
é o disco aberto de centro em e o raio a bola fechada é o disco fechado de centro em e
raio e a esfera é o círculo de centro em e raio No espaço tridimensional, quando a
terminologia coincide com a usual, isto é é a bola aberta de centro em e raio
é a bola fechada de centro em e raio e é a esfera de centro em e raio

WWW.UNINGA.BR 35
ENSINO A DISTÂNCIA

Dado um subconjunto um ponto é ponto interior do


conjunto quando existe tal que Se todo ponto de um conjunto é ponto
interior de então afirmamos que é um conjunto aberto. Portanto, se é um conjunto aberto
e então, existe tal que
Agora, estamos aptos para definir as derivadas parciais. Como sempre, iniciamos com as
funções de duas variáveis:

Definição: Sejam uma função definida em um conjunto aberto


e

i) A derivada parcial de em relação a no ponto é o limite:

desde que exista o limite.


ii) A derivada parcial de f em relação a y no ponto P=(a,b) é o limite:

CÁLCULO II | UNIDADE 2
Como e é um conjunto aberto, existe tal que
Observamos que dado a função
podemos fixar uma das variável e deixar a outra variável livre. Isto define duas funções, a saber,
para e para A função
é a restrição da função ao segmento de reta ,
paralelo ao eixo passando pelo ponto enquanto que a função é a restrição da função
ao segmento de reta , paralelo ao eixo passando
pelo ponto É fácil ver que a derivada da função de uma variável no ponto é dada por:

Analogamente, a derivada da função de uma variável no ponto é dada por:

WWW.UNINGA.BR 36
ENSINO A DISTÂNCIA

Logo, diante da discussão feita, vemos que, na prática, para calcular a derivada parcial
de com relação a , em um ponto derivamos a função de modo usual,
considerando como constante. Para calcular a outra derivada parcial, isto é, a derivada parcial
de com relação a , em um ponto derivamos a função usualmente, mas agora,
considerando como constante. Observe ainda que geometricamente, a derivada parcial
é o coeficiente angular da reta tangente à curva no espaço, obtida pela interseção do gráfico
de com o plano no ponto Analogamente,
a derivada parcial é o coeficiente angular da reta tangente à curva no espaço,
obtida pela interseção do gráfico de com o plano no ponto

CÁLCULO II | UNIDADE 2
Figura 16 - Interpretação geométrica da derivada parcial. Fonte: Stewart (2013).

Observação: Também é comum denotarmos as derivadas parciais como:

Exercício 9:
a) Se calcule as derivadas parciais num ponto genérico

b) Determine e

Solução: Para calcular a derivada parcial com relação a devemos derivar , considerando
como constante. Então:

Analogamente,

WWW.UNINGA.BR 37
ENSINO A DISTÂNCIA

Exercício 10: Seja Calcule:

Solução: Inicialmente, calculamos as derivadas num ponto genérico (x,y):

Portanto,

Observação: Para funções de três ou mais variáveis, procedemos de modo semelhante

CÁLCULO II | UNIDADE 2
para calcular as derivadas parciais, isto é, sempre deixamos uma das variáveis livres e fixamos
todas as outras.

Exercício 11: Calcule as derivadas parciais da função

Solução:

Exercício 12: Calcule as derivadas parciais da função f(x,y,z)= x2 sen(yz).

Solução:

WWW.UNINGA.BR 38
ENSINO A DISTÂNCIA

Podemos, ainda, considerar derivadas parciais de ordem superior, ou seja, derivadas de


segunda ordem, terceira ordem, etc. Por exemplo, se z=f(x,y), uma função definida em um aberto
D(f) ⊂ ℝ2, definimos as derivadas de segunda ordem por:

As derivadas e são ditas derivadas mistas, e quando a função é regular, estas


derivadas serão iguais. Precisamente, considerando as derivadas e são contínuas, então

Outra interpretação geométrica para as derivadas parciais de primeira ordem é a definição


de plano tangente ao gráfico de uma função Considere uma superfície gráfico de
uma função e um ponto . Uma reta é tangente à no ponto se
ela é tangente a alguma curva sobre a superfície passando pelo ponto O plano tangente à
superfície no ponto é o plano que contém todas as retas tangentes à no ponto

CÁLCULO II | UNIDADE 2
Figura 17 - Ilustração geométrica do plano tangente. Fonte: Google Images (2018).

Sem muita dificuldade, podemos demonstrar que se é uma função definida


em um domínio aberto e tem derivadas parciais contínuas em uma vizinhança
de um ponto então a equação do plano tangente ao gráfico de (superfície
no ponto onde é dada por:

Exercício 13: Determine a equação do plano tangente à superfície


no ponto (2,-1,7).

Solução: Observamos que o ponto (2,-1,7) está no gráfico da função pois


Além disso, calculando as derivadas
obtemos e as quais são funções contínuas. Portanto, vemos que
e , e consequentemente a equação do plano tangente é:

WWW.UNINGA.BR 39
ENSINO A DISTÂNCIA

4 - DIFERENCIABILIDADE E A REGRA DA CADEIA


As derivadas parciais são taxas de variação nas direções dos eixos coordenados e não
garantem regularidade da função, pois pode ter derivadas parciais em um ponto e ser descontínua
neste ponto. Para asseguramos a regularidade de uma função, precisamos que ela seja derivável
(ou diferenciável). Assim, definimos:

Definição: Uma função z=f(x,y) definida em um domínio aberto D (f) ⊂ ℝ2 é diferenciável


(ou derivável) em um ponto P=(a,b)∈D(f) se as duas condições são satisfeitas:

i) Existem as derivadas parciais fx (P) e fy (P);

A derivada da função no ponto é a aplicação linear definida por:

CÁLCULO II | UNIDADE 2
O vetor é chamado gradiente da função no ponto Portanto,
a derivada de uma função diferenciável em um ponto é o vetor gradiente neste
ponto, isto é,
Provar a diferenciabilidade de uma função em um ponto não é uma tarefa simples, pois
não é trivial verificar a condição . Entretanto, existem resultados mais “palatáveis” suficientes
para que tenhamos a diferenciabilidade, por exemplo: Se definida em um domínio
aberto possui as derivadas parciais em uma vizinhança de um
e estas são contínuas no ponto então, é diferenciável no ponto Logo, uma função que
possui derivadas parciais contínuas, sempre é diferenciável. O conceito de diferenciabilidade,
para funções três ou mais variáveis, é facilmente obtido, trocando-se o por ou
Como sabemos, desde o estudo do cálculo diferencial para funções de uma variável, a
regra da cadeia expressa gera a derivada da composta de duas funções. Em toda discussão que
segue, nesta seção, vamos admitir funções suaves, ou seja, funções que possuem derivadas de
primeira ordem (funções diferenciáveis) e indicar como calculamos derivadas da composta.

WWW.UNINGA.BR 40
ENSINO A DISTÂNCIA

De modo geral, temos a regra da cadeia:

Exercício 14: Sejam Considere a


função composta e usando a regra da cadeia, calcule as derivadas
e

Solução: Pela regra da cadeia, temos

CÁLCULO II | UNIDADE 2
Então, calculamos cada uma das derivadas:

Substituindo os valores encontrados, temos:

WWW.UNINGA.BR 41
ENSINO A DISTÂNCIA

Exercício 15: Se

determine .

Solução: Aplicando a regra da cadeia temos

5 - DERIVADAS DIRECIONAIS, MÁXIMOS E MÍNIMOS

Sejam uma função definida e diferenciável em um domínio aberto


e um vetor unitário ( ).

CÁLCULO II | UNIDADE 2
Então, a reta que passa pelo ponto e tem a direção do vetor é parametrizada por:

A restrição da função f sobre a reta é dada pela composta (f∘α) e, usando a regra da
cadeia, sua derivada no ponto t = 0 é dada por:

Logo, a expressão acima dá o valor da taxa de variação da função f, na direção do vetor


unitário u, no ponto P, chamada derivada direcional de f, na direção u, no ponto P. Denotamos
a derivada direcional por:

O raciocínio acima se aplica em qualquer espaço n-dimensional, isto é, dada uma função
diferenciável , e um vetor unitário (
), a derivada

WWW.UNINGA.BR 42
ENSINO A DISTÂNCIA

Exercício 16: Dados e calcule a derivada direcional


na direção do vetor

Solução: Inicialmente, vejamos que ou seja, o vetor não é


unitário. Logo, devemos tomar o vetor vetor unitário na mesma direção de Para isto, basta
considerar

Agora, calculamos o gradiente de f:

CÁLCULO II | UNIDADE 2
Exercício 17: Calcule a derivada direcional da função no
ponto na direção do vetor (3,4,-12).

Solução: Se então Portanto


tomamos

Sabemos que a derivada direcional é a taxa de variação da função, no ponto considerado,


na direção de um dado vetor. Usando que o produto escalar de dois vetores pode ser calculado
como produto das normas pelo cosseno do ângulo entre os vetores, obtemos:

WWW.UNINGA.BR 43
ENSINO A DISTÂNCIA

que tem seu valor máximo igual a quando (o que significa


. Concluímos, assim, que dentre todas as possíveis direções, a direção do vetor gradiente é a que
dá o maior valor da derivada direcional. Ou ainda, a direção em que tem a máxima taxa de
variação é a do vetor gradiente .

Exercício 18: Seja qual a direção em que cresce mais


rapidamente, no ponto e encontra a derivada direcional de , nesta direção?

Solução: Pela discussão acima, sabemos que a direção de maior crescimento da função
é a direção do vetor gradiente, e o respectivo valor da derivada direcional é o módulo do vetor
gradiente. Logo:

Portanto, a direção em que f cresce mais rapidamente é a direção do vetor unitário

CÁLCULO II | UNIDADE 2
e

Finalizamos esta seção com um estudo sobre máximos e mínimos locais para funções reais
de duas variáveis reais. Lembramos que por denotamos a bola aberta de centro em e
raio que, no caso bidimensional, é o disco aberto de centro em e raio Seja
uma função definida em um domínio aberto um ponto é um ponto de
máximo local da função se existe tal que para todo
Analogamente, um ponto é um ponto de mínimo local da função se existe
tal que para todo Os pontos de máximos e mínimos locais
de uma função são chamados pontos de extremos relativos. Por outro lado, afirmamos que
é um ponto de máximo global (ou absoluto) da função se para
todo Também, afirmamos que é um ponto de mínimo global (ou
absoluto) da função se para todo
Como no caso unidimensional, os candidatos no ponto máximo ou mínimo local de
uma função diferenciável são os chamados pontos críticos, que, por definição, são os pontos
onde as derivadas parciais se anulam. Assim, de modo análogo ao teste da derivada segunda para
máximos e mínimos locais de função de uma variável, temos o seguinte resultado:

WWW.UNINGA.BR 44
ENSINO A DISTÂNCIA

Teorema: Seja uma função definida em um domínio aberto de


modo que suas derivadas parciais de segunda ordem sejam contínuas em uma vizinhança de um
ponto crítico . Seja o número real dado por:

Exercício 19: Estude os extremos relativos da função

Solução: Inicialmente, encontramos os pontos críticos, ou seja, os pontos onde as


derivadas parciais se anulam. Calculando as derivadas parciais temos:

CÁLCULO II | UNIDADE 2
Assim, devemos resolver o sistema:

Da primeira equação, encontramos que Pela segunda equação, temos que


Portanto, as soluções do sistema são os pontos e Agora, devemos
testar cada um dos pontos críticos. Calculamos as derivadas de segunda ordem:

Logo, o ponto é o ponto de mínimo local e o respectivo valor mínimo local


é

WWW.UNINGA.BR 45
ENSINO A DISTÂNCIA

b) Para o ponto crítico

Logo, o ponto não é ponto de máxima local nem de ponto de mínimo


local.

Em muitas situações aplicadas a problemas práticos, desejamos estudar os extremos de


uma função quando suas variáveis independentes estão sujeitas a uma restrição. Por exemplo:
dentre todos os retângulos que tem perímetro constante igual a quais as dimensões do que
tem maior área? Neste caso, se denotarmos por e as medidas do comprimento e da largura
do retângulo, o problema consiste em maximizar a função área do retângulo
quando e satisfaz a equação Para as funções de duas variáveis,
matematicamente, o problema se apresenta da seguinte forma: dada uma função
determine seus valores, máximo e mínimo, quando as variáveis e satisfazem a uma restrição
descrita por uma equação Isto é:

CÁLCULO II | UNIDADE 2
Problemas deste tipo podem ser resolvidos de diferentes maneiras. Em particular, existe
método bastante interessantes que decorrem de um clássico teorema, chamado teorema dos
multiplicadores de Lagrange, por isto denominado método dos multiplicadores de Lagrange.
Supondo que as funções e sejam regulares (continuamente diferenciáveis), que os valores
extremos existam e que sobre a curva então, os valores máximo e mínimo
de , sujeitos à restrição podem ser determinados pelos seguintes passos:
1o Passo: Determine todos os valores e solução do sistema:

2o Passo: Calcule em todos os pontos obtidos no passo anterior. O maior desses valores
é o valor máximo de , sujeito à restrição, enquanto que o menor dos valores é o valor mínimo
de sujeito à restrição.

WWW.UNINGA.BR 46
ENSINO A DISTÂNCIA

Multiplicadores de Lagrange
http://tvcultura.com.br/videos/33565_calculo-ii-aula-11-multiplicadores-de
lagrange.html

Exercício 20: Dentre todos os retângulos que tem perímetro constante igual a quais as
dimensões do que tem maior área?

Solução: A função é e a restrição é . Podemos


observar que, sujeito à restrição, não existe valor mínimo de , mas existe um valor máximo.
Calculando as derivadas, obtemos:

Logo, pelo método dos multiplicadores, o ponto de máximo (que sabemos que existe) é
obtido e resolvendo o sistema:

CÁLCULO II | UNIDADE 2
Das duas primeiras equações, concluímos que Substituindo isto na terceira
equação, temos que Portanto, e concluímos que o retângulo
de maior área tem perímetro constante igual a é o quadrado de lado

WWW.UNINGA.BR 47
ENSINO A DISTÂNCIA

6 - CONSIDERAÇÕES FINAIS
Neste módulo, vimos que o cálculo diferencial para as funções reais de uma variável,
se estende de modo natural para as funções de duas ou mais variáveis. A novidade essencial
que surge nesta extensão é devido ao aumento da dimensão nos domínios das funções. Este
aumento da dimensão proporciona uma infinidade de maneiras (caminhos) distintas para se
aproximar de um ponto dado, enquanto que no caso unidimensional (na reta real) só tínhamos
as aproximações laterais (pela esquerda e pela direita) do ponto. A noção de taxa de variação
implicou nos conceitos de derivadas parciais e derivadas direcionais, extremamente úteis para o
estudo do comportamento da função nas vizinhanças de um ponto dado no seu domínio.
Neste contexto, um resultado crucial foi observar que o vetor gradiente da função indica a
direção de crescimento máximo e o módulo deste vetor é o valor da taxa máxima de variação. Por
meio do conceito de diferenciabilidade, foi possível aproximar uma função dada, nas vizinhanças
de um ponto dado no domínio, pelo plano tangente e, com isto, obter uma teoria de linearização.
Fechamos o estudo com resultados sobre otimização e o método dos multiplicadores de Lagrange.

CÁLCULO II | UNIDADE 2

WWW.UNINGA.BR 48
UNIDADE ENSINO A DISTÂNCIA

03
DISCIPLINA:
CÁLCULO II

INTEGRAIS MÚLTIPLAS
PROF.A MA. MIRIAM EULALINA MARTINS FROTA

SUMÁRIO DA UNIDADE

INTRODUÇÃO ........................................................................................................................................................... 50
1 - A INTEGRAL DUPLA ............................................................................................................................................ 51
2 - A INTEGRAL DUPLA EM COORDENADAS POLARES (MUDANÇA DE VARIÁVEL) ....................................... 60
3 - A INTEGRAL TRIPLA .......................................................................................................................................... 63
4 - A INTEGRAL TRIPLA EM COORDENADAS CILÍNDRICAS E ESFÉRICAS ....................................................... 67
5 - CONSIDERAÇÕES FINAIS .................................................................................................................................. 72

WWW.UNINGA.BR 49
ENSINO A DISTÂNCIA

INTRODUÇÃO
Como sabemos, o cálculo integral para funções de uma variável teve sua origem com
o problema de se calcular áreas de regiões planas não regulares. Indo mais além, descobrimos
sua utilidade em diversos outros problemas como, por exemplo, no cálculo de volumes de
sólidos de revolução, comprimento de arcos, atração gravitacional, distância percorrida por um
móvel a partir de sua velocidade, cálculo da média de uma distribuição contínua de dados, etc.
Neste módulo, faremos o estudo da integral definida para funções de duas ou mais variáveis.
Inicialmente, como já fizemos no módulo II, introduzimos o conceito para funções de duas
variáveis, dando origem à integral dupla. Depois, em três variáveis independentes teremos a
integral tripla. Também veremos importantes mudanças de coordenadas.

CÁLCULO II | UNIDADE 3

WWW.UNINGA.BR 50
ENSINO A DISTÂNCIA

1 - A INTEGRAL DUPLA

A título de motivação, consideremos uma função não negativa definida em


um domínio limitado Então, temos que para todo e
está contido num retângulo O gráfico da função é uma superfície em eo
problema que se apresenta é como definir (ou calcular) o volume do sólido formado pela região
abaixo da superfície (gráfico de e acima do domínio ?

CÁLCULO II | UNIDADE 3
Figura 1 - Volume do sólido formado pela região abaixo da superfície (gráfico de e acima do domínio
. Fonte: Google Images (2018).

A solução do problema proposto é obtida da seguinte forma: note que


então dividimos o retângulo que
contém o domínio em subretângulos obtidos pelo produto cartesiano de uma
partição do intervalo com uma partição do intervalo Nestes termos, afirmamos que
é uma partição do retângulo e a norma desta partição é o número positivo
, onde é a medida da diagonal do sub-retângulos

WWW.UNINGA.BR 51
ENSINO A DISTÂNCIA

Figura 2 - Partição do retângulo . Fonte: Google Images (2018).

Em cada um dos sub-retângulos escolhemos aleatoriamente um ponto


e representamos por a área do retângulo Estendemos a função a todo retângulo

CÁLCULO II | UNIDADE 3
pondo para todos os pontos de então,
sempre que Assim, é o volume do paralelepípedo de base
e altura , que se anula quando o ponto Portanto, uma aproximação para
o volume do sólido é a soma:

e, esta aproximação torna-se cada vez melhor à medida em que o número de sub-
retângulos aumenta (equivalentemente ). Logo, é razoável definirmos o volume do
sólido como sendo o limite das somas quando ou seja:

A solução do problema motiva a definição.

Definição: Dada uma função definida em um domínio limitado


, se as somas ficam arbitrariamente próximas de um número
real L, à medida em que , independentemente da escolhas dos pontos
afirmamos que é integrável sobre e sua integral dupla sobre é o número Escrevemos:

WWW.UNINGA.BR 52
ENSINO A DISTÂNCIA

Observação 1: As somas são chamadas somas de


Riemann da função associadas à partição

Observação 2: Em particular, quando a função constante é igual a um no domínio


limitado isto é, para todo é integrável sobre , afirmamos que é
mensurável (ou tem área) e o valor da sua área é a integral dupla:

A seguir, enumeramos uma série de propriedades da integral dupla as quais podem ser
demonstradas a partir da definição supracitada. No que segue, sempre estamos considerando
como uma função definida em um domínio mensurável .

CÁLCULO II | UNIDADE 3
1- Se é integrável em então é limitada em

2- Se e são funções integráveis em e é uma constante, então e são


funções integráveis em e vale as igualdades:

3- Se e são funções integráveis em e então:

4- Se é contínua em um retângulo então é integrável sobre


qualquer subconjunto mensurável

WWW.UNINGA.BR 53
ENSINO A DISTÂNCIA

5- Sejam e subconjuntos limitados e mensuráveis, tais que e a área


. Se é integrável em então é integrável em e também em e a seguinte
igualdade é satisfeita:

O cálculo da integral dupla, via limite das somas de Riemann, é uma tarefa bastante
complicada e tecnicamente inviável. Durante muito tempo, esta foi a única alternativa quando
se desejava calcular uma integral dupla, até que em 1907, um resultado, devido ao matemático
italiano Guido Fubini, facilitou este cálculo, encontrando, assim, um caminho alternativo por
meio de integrais iteradas.

Teorema (de Fubini): Seja uma função integrável em um retângulo

Se para cada existe a integral então,

existe, também, a integral e vale a igualdade:

CÁLCULO II | UNIDADE 3
Analogamente, se para cada existe a integral então,

existe também a integral e vale a igualdade:

Exercício 1: Calcule a integral dupla da função no retângulo

Solução: Usando o teorema de Fubini, temos:

WWW.UNINGA.BR 54
ENSINO A DISTÂNCIA

Se no cálculo acima fizermos primeiro a integral em relação a y e depois à integral


em relação a x, encontramos o mesmo resultado para a integral dupla (faça os
cálculos).

Exercício 2: Determine o volume do sólido limitado acima, pelo paraboloide


; abaixo pelo plano coordenado ; e nas laterais, pelos planos e
os planos coordenados e

Solução: Considere o retângulo Então, o volume do sólido descrito é


dado pela integral dupla

CÁLCULO II | UNIDADE 3
O Teorema de Fubini pode ser estendido para se calcular a integral dupla de uma função
sobre regiões mais gerais do que retângulos, especificamente, regiões compreendidas entre
gráficos de funções contínuas. Precisamente, sejam duas funções contínuas com
para todo
, e considere

Figura 3 - Região compreendida entre gráficos de funções contínuas. Fonte: o autor.

WWW.UNINGA.BR 55
ENSINO A DISTÂNCIA

Se uma função integrável em e para todo existe a integral

então, existe a integral e vale a igualdade:

Exercício 3: Calcule a integral dupla da função sobre a região


limitada pela parábola e a reta

Solução: Vejamos, inicialmente, as interseções da reta com a parábola. Para isto,


resolvemos a equação:

Na figura, fazemos um esboço da região

Figura 4 - Esboço da região D. Fonte: o autor. CÁLCULO II | UNIDADE 3

Portanto, a integral é calculada como segue:

WWW.UNINGA.BR 56
ENSINO A DISTÂNCIA

Exercício 4: Usando a integral dupla, encontre a área da região plana limitada pelas
parábolas e

Solução: Vejamos as interseções das duas parábolas. Para isto, resolvemos a equação:

Portanto, os pontos de interseção são (0,0) e (2,4). A seguir, apresentamos um esboço da


região

CÁLCULO II | UNIDADE 3

Figura 5 - Esboço da região Ω. Fonte: o autor.

Então, a área de Ω, via integral dupla, é:

WWW.UNINGA.BR 57
ENSINO A DISTÂNCIA

De modo análogo, podemos tratar a integral dupla sobre regiões do tipoD

em que, são duas funções contínuas com

Figura 6 - Região compreendida entre gráficos de funções contínuas. Fonte: o autor.

CÁLCULO II | UNIDADE 3
Neste caso, a integral dupla sobre a região D é dada por:

Exercício 5: Calcule o volume do sólido abaixo do plano e acima do círculo


no plano

Solução: Se é o volume do sólido indicado, temos:

Exercício 6: Considere a integral dupla onde é a região plana limitada pela


reta e a parábola Indique as integrais iteradas para as duas ordens de
integração e calcule o valor da integral dupla usando a ordem de integração mais fácil.

WWW.UNINGA.BR 58
ENSINO A DISTÂNCIA

Solução: Para que tenhamos um esboço da região vamos determinar os pontos de


interseção da reta com a parábola. Assim, resolvemos a equação:

e os pontos de interseção são (1,-1) e (4,2). Na figura temos um esboço da região D:

CÁLCULO II | UNIDADE 3
Figura 7 - Esboço da região D. Fonte: o autor.

As integrais iteradas são dadas por:

ou:

É claro que a segunda opção é mais fácil de ser resolvida. Então,

WWW.UNINGA.BR 59
ENSINO A DISTÂNCIA

2 - A INTEGRAL DUPLA EM COORDENADAS POLARES


(MUDANÇA DE VARIÁVEL)
No cálculo I, umas das principais técnicas de integração, para o cálculo de integrais de
funções de uma variável, é a integração por substituição, que nada mais é do que uma fórmula de
mudança de variável, isto é:

onde é uma mudança de variável que transforma o intervalo


no intervalo Para integrais duplas, integrais de funções de duas variáveis, existe uma
fórmula análoga dada por:

CÁLCULO II | UNIDADE 3
onde, é uma mudança de variável que transforma
a região na região isto é, e é o jacobiano de no ponto definido
por:

Uma mudança de variável muito útil é a mudança para coordenadas polares, definida
por:

Cujo jacobiano é Então, aplicando a fórmula acima temos

WWW.UNINGA.BR 60
ENSINO A DISTÂNCIA

Para maiores informações sobre coordenadas polares consulte STEWART, J.;


Cálculo. Vol. 2, 7a. Ed. São Paulo: Editora Cengage Learning, 2013.

Exercício 7: Calcule a integral dupla onde é a região no primeiro


quadrante limitada pelo círculo

Solução: Consideramos a mudança de coordenadas

que transforma o retângulo polar no quarto

CÁLCULO II | UNIDADE 3
do disco situado no primeiro quadrante, isto é, Então, fazendo a
mudança na integral, temos:

Exercício 8: Calcule o volume do sólido compreendido entre os dois cilindros centrados


na origem de raios iguais a 1 e 2, respectivamente, no primeiro octante, limitado acima pelo
paraboloide e abaixo pelo plano

Solução: O volume do sólido descrito no exercício é dado pela integral da função


sobre a região , no primeiro quadrante limitada pelos círculos
, e os eixos coordenados.

WWW.UNINGA.BR 61
ENSINO A DISTÂNCIA

Figura 8 - Esboço da região D. Fonte: o autor

CÁLCULO II | UNIDADE 3
Exercício 9: Seja a metade do círculo de centro no ponto raio igual a 1, situada no
primeiro quadrante, isto é, Calcule a integral

dupla .

Solução: Usando coordenadas polares podemos descrever a


região como segue. A equação do círculo em coordenadas polares é:

em que, para encontramos Logo, a integral fica:

WWW.UNINGA.BR 62
ENSINO A DISTÂNCIA

3 - A INTEGRAL TRIPLA

De modo análogo ao estudo da integral dupla, quando queremos definir a integral de


uma função de três variáveis, sobre uma região do espaço euclidiano chegamos ao conceito
de integral tripla.
Suponhamos que é uma função definida em um domínio limitado
contido em um paralelepípedo Então, dividimos o
paralelepípedo que contém o domínio em subparalelepípedos obtidos pelo
produto cartesiano de uma partição do intervalo com uma partição do intervalo e
uma partição do intervalo Nestes termos, afirmamos que é uma partição
do paralelepípedo e a norma desta partição é o número positivo , em que é
a medida da diagonal do paralelepípedo Em cada um dos subparalelepípedos escolhemos
aleatoriamente um ponto e representamos por o volume do paralelepípedo
Estendemos a função a todo paralelepípedo pondo

CÁLCULO II | UNIDADE 3
para todos os pontos de então, sempre que Desta forma,
definimos a soma de Riemann da função associada à partição por:

Definição: Dada uma função definida em um domínio limitado


, se as somas de Riemann ficam arbitrariamente próximas
de um número real L, à medida em que , independentemente da escolhas dos pontos
dizemos que é integrável sobre e sua integral tripla sobre é o número
Escrevemos:

WWW.UNINGA.BR 63
ENSINO A DISTÂNCIA

Observação: Em particular, quando a função constante é igual a um no domínio


limitado isto é, para todo é integrável sobre afirmamos que
é mensurável (ou tem volume) e o valor do seu volume é a integral tripla

Observando as adaptações naturais, devido à passagem de duas dimensões para


três dimensões, todas as propriedades válidas para a integral dupla continuam
válidas para a integral tripla. Isto é, cada uma das propriedades para a integral
dupla tem sua versão correspondente (análoga) para a integral tripla.

Merece destaque o teorema de Fubini, que na versão tridimensional, indica que o cálculo
de uma integral tripla pode ser feito por meio de integrais iteradas, neste caso, três integrais
simples. No caso do domínio da função ser um paralelepípedo D=[a,b]×[c,d]×[u,v], então:

CÁLCULO II | UNIDADE 3
Na expressão acima integramos primeiro em relação a x, depois em relação a y e em
relação a z. Existem, ainda, outras cinco possíveis ordens de integração, todas dando o mesmo
resultado.

Exercício 10: Calcule a integral tripla onde é o paralelepípedo dado


por

Solução:

WWW.UNINGA.BR 64
ENSINO A DISTÂNCIA

Podemos considerar regiões mais gerais do que paralepípedos. Suponha que a região de
integração seja limitada pelo gráfico de duas funções contínuas e
definidas em um domínio limitado isto é,

Então, a integral tripla sobre é dada por:

Exercício 11: Faça um esboço do sólido limitado pelos planos coordenados e pelo

plano e calcule a integral tripla

Solução: Os traços do plano são dados por:

a) z=0⇒x+y=1 (uma reta no plano xy)


b) x=0⇒y+z=1 (uma reta no plano yz)

CÁLCULO II | UNIDADE 3
c) y=0⇒x+z=1 (uma reta no plano xz)

Traçando as retas, obtemos o esboço do sólido D:

Figura 9 - Esboço do sólido D. Fonte: o autor.

WWW.UNINGA.BR 65
ENSINO A DISTÂNCIA

Podemos ver que o sólido é a região limitada pelos gráficos das funções
e definidas no triângulo limitado pelos eixos coordenados e e a reta
Portanto, a integral tripla fica:

Exercício 12: Usando a integral tripla, determine o volume do sólido limitado pela
superfície y=x2 e os planos y+z=4 e z=0.

Solução: O volume do sólido é dado pela integral tripla

CÁLCULO II | UNIDADE 3
De modo análogo, podemos considerar que a região de integração seja limitada pelo
gráfico de duas funções contínuas e definidas em um domínio
limitado isto é:

então, a integral tripla sobre é dada por:

WWW.UNINGA.BR 66
ENSINO A DISTÂNCIA

Também há o caso em que a região de integração é limitada pelo gráfico de duas funções
contínuas e definidas em um domínio limitado isto é,
e, nesse caso, a integral tripla
sobre é dada por:

4 - A INTEGRAL TRIPLA EM COORDENADAS


CILÍNDRICAS E ESFÉRICAS
A mudança de variável na integral tripla, analogamente ao caso tratado na integral dupla,
está estabelecida pela fórmula:

CÁLCULO II | UNIDADE 3
onde é uma mudança de
variável que transforma a região na região isto é, e é o jacobiano
de no ponto definido por:

A mudança de variável para coordenadas cilíndricas é a transformação

cujo jacobiano é

WWW.UNINGA.BR 67
ENSINO A DISTÂNCIA

Portanto, a fórmula de mudança de variável da integral tripla, em coordenadas cilíndricas


é dada por:

Exercício 13: Aplique a fórmula de mudança de variáveis em coordenadas cilíndricas e

calcule a integral

Solução: Analisando os limites de integração, vemos que a região de integração é o sólido


limitado pelo paraboloide , acima do plano Note que a projeção desse
paraboloide sobre o plano é o disco Fazendo a mudança de coordenadas, para
coordenadas cilíndricas, temos que a equação do paraboloide fica enquanto que o
disco sobre o plano fica e Logo efetuando a mudança temos:

CÁLCULO II | UNIDADE 3

WWW.UNINGA.BR 68
ENSINO A DISTÂNCIA

Exercício 14: Seja o sólido limitado lateralmente pelo cilindro acima


pelo paraboloide e abaixo pelo plano Suponha que a densidade de
massa em qualquer ponto de seja proporcional à distância do ponto ao eixo do cilindro.
Calcule a massa do sólido

Solução: Sabemos que a massa é a integral da função densidade de massa, sobre o


domínio solido Se é a função densidade de massa, desde que seu valor em
qualquer ponto de é proporcional à distância do ponto ao eixo do cilindro, devemos ter que
, em que é uma constante de proporcionalidade. Portanto, a massa
do sólido é

Fazendo a mudança de coordenadas, de retangulares para cilíndricas:

CÁLCULO II | UNIDADE 3
a equação do paraboloide fica a equação do cilindro fica e o sólido
, em coordenadas cilíndricas, é descrito por:

Então:

Para determinadas regiões, os cálculos ficam mais fáceis quando usamos coordenadas
esféricas, especialmente as regiões ditas “típicas esféricas” (partes de uma esfera), cujas equações se
tornam extremamente simples quando descritas nestas coordenadas. Nestes casos, é conveniente
mudarmos de coordenadas cartesianas para coordenadas esféricas. A transformação é definida
por:

WWW.UNINGA.BR 69
ENSINO A DISTÂNCIA

Figura 11 - Coordenadas Esféricas. Fonte: o autor.

cujo jacobiano é Portanto, a fórmula de mudança de variável


da integral tripla, em coordenadas cilíndricas, é dada por:

CÁLCULO II | UNIDADE 3
Exercício 15: Calcule o volume de uma esfera de raio

Solução: Sem perda de generalidade, consideramos a esfera de centro na origem, cuja


equação, em coordenadas cartesianas é dada por Usando, ainda, a simetria
da região, podemos calcular o volume da esfera no primeiro octante e multiplicar o resultado por
oito. Portanto, em coordenadas cartesianas, temos que

WWW.UNINGA.BR 70
ENSINO A DISTÂNCIA

Para o cálculo da integral acima, fazemos a mudança de coordenadas cartesianas para


coordenadas esféricas:

Exercício 16: Calcule a integral tripla onde é o sólido compreendido


entre as esferas e

CÁLCULO II | UNIDADE 3
Solução: O sólido descrito em coordenadas esféricas fica

Então,

WWW.UNINGA.BR 71
ENSINO A DISTÂNCIA

Ao final deste módulo, o vídeo apresentado no link: <http://tvcultura.com.br/


videos/33445_calculo-ii-aula-19-mudanca-de-variaveis-em-integrais-multiplas.
html> complementa os conteúdos abordados.

5 - CONSIDERAÇÕES FINAIS
A motivação central para o conceito da integral dupla foi o cálculo do volume de sólidos
irregulares, situação análoga ao caso unidimensional, em que a motivação para a integral simples
foi o cálculo da área de regiões planas irregulares. Neste contexto, a integral dupla se apresenta
como limite das somas de Riemann, quando a norma da partição tende a zero. O cálculo da integral
dupla, por meio do limite das somas de Riemann, tecnicamente, apresenta sérias dificuldades e
o Teorema de Fubinni, sobre integrais iteradas, juntamente com o Teorema Fundamental do
Cálculo, facilitam este cálculo. A teoria de integração dupla se amplia, muito naturalmente, para

CÁLCULO II | UNIDADE 3
as funções de três variáveis, definidas em regiões do espaço tridimensional, dando origem ao
conceito da integral tripla.
Quando consideramos regiões planas “circulares”, como: círculos, setores circulares,
arruelas ou partes de arruelas, é extremamente conveniente considerar as coordenadas polares.
Por isso, obtivemos um resultado sobre mudança de variável na integral e descrevemos a fórmula
da integral dupla em coordenadas polares. No caso da integral tripla, quando integramos sobre
regiões limitadas por cilindros ou partes de cilindros e esferas, as coordenadas convenientes são
as coordenadas cilíndricas ou as coordenadas esféricas. O resultado principal nesta direção são as
fórmulas da integral tripla em coordenadas cilíndricas e em coordenadas esféricas.

WWW.UNINGA.BR 72
UNIDADE ENSINO A DISTÂNCIA

04
DISCIPLINA:
CÁLCULO II

CÁCULO VETORIAL E OS
TEOREMAS DE DIVERGÊNCIA
PROF.A MA. MIRIAM EULALINA MARTINS FROTA

SUMÁRIO DA UNIDADE

INTRODUÇÃO ........................................................................................................................................................... 74
1 - CURVAS ................................................................................................................................................................ 75
2 - A INTEGRAL DE LINHA DE UM CAMPO VETORIAL ........................................................................................ 81
3 - TEOREMA DE GREEN ........................................................................................................................................ 88
4 - SUPERFÍCIES ...................................................................................................................................................... 91
5 - INTEGRAIS DE SUPERFÍCIE ............................................................................................................................. 96
6 - TEOREMA DA DIVERGÊNCIA E TEOREMA DE STOKES ............................................................................... 100
7 - CONSIDERAÇÕES FINAIS ................................................................................................................................ 105

WWW.UNINGA.BR 73
ENSINO A DISTÂNCIA

INTRODUÇÃO
Nos dois módulos anteriores, nosso tema de estudo foi o cálculo diferencial e integral para
funções reais de duas ou mais variáveis, chamadas de campos escalares. Neste módulo, estudamos
o cálculo para campos vetoriais, que são funções vetoriais que a cada ponto do plano (ou do
espaço) associam um vetor. Devemos definir as integrais de linha e as integrais de superfície
e apresentar os clássicos teoremas de divergência, conhecidos na literatura como Teorema de
Green, Teorema de Gauss e Teorema de Stokes.

CÁLCULO II | UNIDADE 4

WWW.UNINGA.BR 74
ENSINO A DISTÂNCIA

1 - CURVAS
Um caminho no plano, ou uma curva plana, é uma função vetorial, de uma variável real,
definida em um intervalo isto é, uma função do tipo:

Afirmamos que a curva é lisa quando a função é continuamente diferenciável


( é derivável com a derivada contínua) e O conjunto imagem
é chamado o traço da curva e este, de modo natural,
possui um sentido (uma orientação) que é aquele quando este conjunto é percorrido no sentido
crescente dos valores de O ponto é dito o ponto inicial da curva e o ponto é
chamado ponto final da curva Quando os pontos inicial e final coincidem, isto é, quando
afirmamos que a curva é fechada. Um ponto no traço de uma curva é dito
um ponto múltiplo de quanto existem com e tais que
e a curva é denominada curva simples quando seu traço não possui ponto
múltiplo. O comprimento da curva é o número real

CÁLCULO II | UNIDADE 4

Figura 1 - Curvas. Fonte: o autor.

WWW.UNINGA.BR 75
ENSINO A DISTÂNCIA

Um resultado muito interessante, conhecido da literatura como Teorema de Jordan,


garante que toda curva contínua, fechada e simples divide o plano em duas regiões abertas e
disjuntas, sendo uma limitada e outra não-limitada. Enfatizamos que, por definição, uma curva
é uma função e seu traço é um subconjunto do plano, portanto, objetos de naturezas diferentes,
entretanto, quando não há possibilidade de confusão. Por um abuso de linguagem, é comum
não fazer distinção entre a curva e o seu traço, mencionando o traço como sendo uma curva e
afirmar que , é uma parametrização de

Observação: De modo análogo, um caminho no espaço ou uma curva no espaço


euclidiano é uma função vetorial

No mais, tudo que foi dito para uma curva plana estende-se de forma análoga para as
curvas no espaço, ou até mesmo, em qualquer espaço euclidiano

Exemplo 1: A curva plana é uma curva plana lisa


pois O traço de é o segmento de reta que liga o ponto

CÁLCULO II | UNIDADE 4
ao ponto é uma curva lisa, não fechada e simples.

Figura 2 - Traço de . Fonte: o autor.

WWW.UNINGA.BR 76
ENSINO A DISTÂNCIA

Exemplo 2: A curva definida por:

com , que é uma curva lisa no espaço e seu traço é a reta que passa pelo
ponto e tem a direção do vetor

Exemplo 3: A curva é uma curva plana


lisa, não-fechada e simples. Seu traço é o arco de parábola com

CÁLCULO II | UNIDADE 4
Figura 3 - Traço de . Fonte: o autor.

Exemplo 4: O gráfico de uma função continuamente diferenciável


em um intervalo é uma curva lisa em que pode ser naturalmente parametrizada pela
aplicação dada por Neste caso, temos que e
o comprimento de fica

Exemplo 5: A curva é uma curva plana lisa, fechada


e simples. O traço de é o círculo de centro na origem e raio igual a 1, ( orientado
no sentido anti-horário.

WWW.UNINGA.BR 77
ENSINO A DISTÂNCIA

Figura 4 - Traço de γ. Fonte: o autor.

Observação: As curvas

CÁLCULO II | UNIDADE 4
tem o mesmo traço que a curva do exemplo 4. Com isto, chamamos a atenção para o
fato de que curvas diferentes podem ter o mesmo traço. Observe, ainda, que as curvas e tem
comprimento enquanto que a curva tem comprimento Também notamos que as curvas
e tem o mesmo ponto inicial , e a curva tem o ponto inicial As curvas e são
percorridas no sentido anti-horário, enquanto que a curva é percorrida no sentido horário. As
curvas e são simples e a curva não é simples.

Um fato interessante é que dada qualquer curva lisa é sempre


possível encontrar uma outra curva lisa que possui o mesmo traço
e mesma orientação que a curva e satisfaz Afirmamos que
é a parametrização pelo comprimento de arco. É possível ver que onde
é a função comprimento de arco.

WWW.UNINGA.BR 78
ENSINO A DISTÂNCIA

Figura 5 - Ilustração da parametrização pelo comprimento de arco. Fonte: o autor.

Uma vez dado um traço de uma curva lisa, podemos obter diferentes parametrizações
(funções continuamente diferenciáveis) cujo traço é No conjunto de todas as possíveis

CÁLCULO II | UNIDADE 4
parametrizações de afirmamos que duas parametrizações e são
equivalentes, e escrevemos quando existe uma função continuamente diferenciável:

tal que:

WWW.UNINGA.BR 79
ENSINO A DISTÂNCIA

Figura 6 - Parametrizações equivalentes. Fonte: o autor.

É possível mostrar que parametrizações equivalentes de determinam o mesmo


comprimento, ou seja, se Por exemplo,

CÁLCULO II | UNIDADE 4
são parametrizações equivalentes do quarto de círculo de centro na origem e raio igual
a 1, no primeiro quadrante. Por outro lado, conforme comentamos no exemplo 5 e a observação
logo após, podemos concluir que:

são parametrizações não equivalentes do círculo unitário de centro na origem.

Um estudo completo e clássico sobre curvas pode ser encontrado no capítulo 1.


Fonte: CARMO. M. P. Geometria Diferencial de Curvas e Superfícies. Sociedade
Brasileira de Matemática – Rio de Janeiro, 2008.

WWW.UNINGA.BR 80
ENSINO A DISTÂNCIA

2 - A INTEGRAL DE LINHA DE UM CAMPO VETORIAL


Agora, podemos definir a integral de um campo vetorial sobre uma curva.

Definição: Sejam , com um


campo vetorial contínuo definido em uma região de e uma curva lisa em
parametrizada por uma função continuamente diferenciável definida por
Assim, o produto escalar:

uma função real contínua definida para todo e, portanto, integrável. Por
definição, a integral:

CÁLCULO II | UNIDADE 4
é denominada a integral de linha do campo ao longo da curva , usualmente denotada

por

É claro que esta definição pode ser facilmente generalizada para qualquer dimensão. Se
é um campo contínuo em uma região e é uma curva suave em
parametrizada por uma função continuamente diferenciável definida por
então a integral de linha do campo ao longo da curva
é

Em particular, se , com
e uma curva lisa em parametrizada por definida por
então:

WWW.UNINGA.BR 81
ENSINO A DISTÂNCIA

É claro que a integral de linha de um campo F ao longo de uma curva C, depende


da parametrização γ escolhida, entretanto, seu valor não se altera quando
consideramos parametrizações equivalentes.

Existe uma importante interpretação física para a integral de linha. Se o campo vetorial
representa um campo de força na região então, o trabalho realizado,
devido à ação deste campo, para deslocar uma partícula, ao longo de uma curva suave, em
é a integral de linha do campo sobre a curva, ou seja:

CÁLCULO II | UNIDADE 4
onde , é uma parametrização
continuamente diferenciável da curva

Exercício 1: Seja um campo de vetores em todo e o quarto do


círculo situado no primeiro quadrante, orientado no sentido anti-horário. Calcule a integral de

linha .

Solução: Uma parametrização para o quarto do círculo, no primeiro quadrante, é


Então:

Portanto,

WWW.UNINGA.BR 82
ENSINO A DISTÂNCIA

Exercício 2: Dado o campo vetorial e a curva


para calcule a integral de linha do campo ao longo da curva

Solução:

Exercício 3: Calcule:

CÁLCULO II | UNIDADE 4
onde é a curva parametrizada por para

Solução:

Exercício 4: Considere o campo vetorial no plano dado por


Calcule, a integral do campo ao longo da reta a partir da origem para o ponto

Solução: Uma parametrização para a reta é para Além disso,

Portanto,

WWW.UNINGA.BR 83
ENSINO A DISTÂNCIA

É claro que nosso estudo pode ser facilmente estendido para curvas lisas por partes, ou
seja, curvas que são a união de um número finito de curvas lisas. Se
onde cada um dos é uma curva lisa de modo que o ponto final de é o ponto inicial de
então:

Exercício 5: Calcule a integral de linha onde é a curva lisa por


partes formada pela parábola , entre os pontos e e o segmento de reta de
para conforme a figura:

CÁLCULO II | UNIDADE 4
Figura 7 - Ilustração da curva . Fonte: o autor.

Solução: A curva é a reunião das duas curvas e cujas parametrizações são dadas
por:

Assim, temos:

WWW.UNINGA.BR 84
ENSINO A DISTÂNCIA

A integral de linha de um campo vetorial é a generalização do conceito de integral para


funções vetoriais, então, é razoável imaginar que haja algum resultado análogo ao teorema
fundamental do cálculo. De fato, existe um resultado nesta direção, que afirma que se um campo
contínuo admite uma função potencial, então, a integral de linha do campo, ao longo de uma
curva, tem uma expressão bem simples em termos da função potencial. Em termos precisos,
afirmamos que uma função escalar diferenciável é um potencial de campo
vetorial quando satisfaz em É claro que se é um potencial de
então, para toda constante , também é um potencial de Quando é um campo
vetorial contínuo em um domínio que possui uma função potencial, afirmamos que é um
campo gradiente em
É possível provar que se é um domínio aberto e conexo; é um campo
gradiente; e são dois pontos em e ( ) uma curva lisa por
partes ligando o ponto ao ponto , então:

onde é um potencial de Observamos que, nas condições acima, a

CÁLCULO II | UNIDADE 4
integral de linha do campo ao longo da curva é independente do caminho que liga o ponto
ao ponto . O valor da integral depende somente dos pontos final e inicial. Reciprocamente,
também é possível mostrar que se é um campo contínuo em um domínio

aberto e conexo tal que a integral de linha é independente do caminho, então, existe
um potencial de em ou seja, é um campo gradiente em Resumindo:

a integral de linha é independente do caminho em se, e somente se, é um campo


gradiente.

Observação: Em Física, quando a integral de linha um campo vetorial é independente do


caminho em , se diz que o campo é conservativo. Logo, um campo contínuo é conservativo
se, e somente se, é um campo gradiente.

Como já foi exposto anteriormente, tudo que estabelecemos para campos vetoriais
em duas dimensões (F:D⊂ℝ2→ℝ2) continuas é válido para campos vetoriais em ℝ3
e até mesmo em ℝn para todo n∈ℕ.

WWW.UNINGA.BR 85
ENSINO A DISTÂNCIA

Exercício 6: Um campo de força muito frequente em Física é o campo gravitacional, que


determina a ação (força) gravitacional do planeta Terra sobre um corpo de massa dado por

, definido por para todo


onde é uma constante positiva, é a massa do corpo, é a massa da Terra e éa
aceleração da gravidade. Em termos das funções componentes:

a) Verifique para é um potencial de

b) Calcule o trabalho realizado pelo campo gravitacional para mover um objeto de


massa do ponto para o ponto .

Solução:
a) A função está definida por

CÁLCULO II | UNIDADE 4
Calculando a derivada parcial em relação a x, temos:

De modo análogo, obtemos as derivadas parciais com relação a y e a z. Assim,

b) O trabalho realizado pelo campo gravitacional para mover um objeto de massa do

ponto para o ponto é dado pela integral de linha


onde é uma curva lisa por partes de para Desde que é um campo gradiente, a integral
de linha é independente do caminho. Logo,

É fácil ver que se é um campo gradiente em um domínio e em que é uma curva

lisa por partes e fechada em então, a integral de linha Reciprocamente, pode-


se provar que se a integral de linha de um campo contínuo em um domínio ao longo de
qualquer curva lisa por partes e fechada em , é igual a zero, então, a integral é independente do
caminho. Resumimos os resultados no teorema que veremos a seguir.

WWW.UNINGA.BR 86
ENSINO A DISTÂNCIA

Teorema: Sejam um subconjunto aberto e conexo e um campo vetorial


contínuo em Então, as seguintes afirmações são equivalentes:
i) é um campo gradiente;

ii) A integral de linha do campo , ao longo de qualquer curva lisa por partes e fechada
em é igual a zero.

iii) Sempre que e são dois pontos em a integral de linha do campo de para
é independente do caminho ( é um campo conservativo).

Observação: No caso bidimensional, isto é, para campos de vetores no plano, há, ainda,
uma equivalência adicional para campos conservativos. Ou seja:

Teorema: Seja um conjunto aberto, limitado e simplesmente conexo


(“sem buracos”) e , definido por um campo
continuamente diferenciável (de classe ) em Assim, é um campo conservativo se, e

somente se,

CÁLCULO II | UNIDADE 4
Exercício 7:
a) Verifique se o campo é um campo conservativo e, em caso
afirmativo, encontre um potencial de

b) Calcule a integral de linha onde é uma curva lisa por


partes, saindo do ponto até o ponto

Solução:
a) Denotamos as funções coordenadas do campo por:

Calculando as derivadas temos:

Então, desde que resulte que o campo é

conservativo em todo o plano Para encontrar uma função potencial devemos ter:

WWW.UNINGA.BR 87
ENSINO A DISTÂNCIA

Integrando a primeira equação em relação à temos onde


é uma função arbitrária. Derivando com relação a e usando a segunda equação do sistema
acima, encontramos:

Logo, é um potencial do campo

b) No item anterior, vimos que é um campo conservativo, então, a integral de linha do


campo é independente do caminho, ou seja, só depende dos valores do potencial nos pontos
inicial e final da curva. Portanto,

CÁLCULO II | UNIDADE 4
3 - TEOREMA DE GREEN
O Teorema de Green é o resultado correspondente ao teorema fundamental do cálculo
para integrais duplas. A grosso modo, ele relaciona o cálculo da integral de linha de um campo
bidimensional, ao longo de uma curva fechada simples , com uma integral dupla sobre a região
delimitada pela curva
Seja um domínio cuja fronteira é um número finito de curva lisa por partes,
fechadas e simples, orientadas no sentido positivo (que deixa a região sempre à esquerda). Se
é um campo de classe em então,

A primeira utilidade do teorema de Green é simplificar o cálculo de determinadas


integrais de linha, cujo valor pode ser obtido mais facilmente pelo cálculo da integral dupla.
Por outro lado, às vezes o melhor é o contrário, ou seja, em determinadas situações temos que o
cálculo da integral dupla é mais complicado do que o cálculo da integral de linha.

WWW.UNINGA.BR 88
ENSINO A DISTÂNCIA

Exercício 8: Calcule a integral de linha onde é o campo vetorial dado


por e é o triângulo de vértices e

Solução: Observamos que ou seja, e


. Aplicando o teorema de Green, temos que:

Claro que poderíamos ter calculado a integral de linha diretamente, sem usar o teorema
de Green. Entretanto, o cálculo seria mais longo. Como a curva é um triângulo, deveríamos
parametrizar cada um dos lados e fazer as três integrais e depois soma-las. Faça os cálculos e
comprove o mesmo resultado.

CÁLCULO II | UNIDADE 4
Exercício 9: Se e é o disco de centro na origem (0,0) e raio

igual a 1, orientado no sentido anti-horário, calcule a integral de linha

Solução: Aplicando o teorema de Green e depois fazendo a mudança para coordenadas


polares, temos:

Outra interessante aplicação do teorema de Green é o cálculo de áreas de regiões planas.


Sabemos que se é uma região limitada, então, a área de é dada por

WWW.UNINGA.BR 89
ENSINO A DISTÂNCIA

Logo, podemos escolher as coordenadas e de um campo vetorial tal que

Por exemplo, tomando:

ou ainda,

Logo, aplicando o teorema de Green, resulta-se que:

Exercício 10: Usando o teorema de Green, encontre a área da região plana delimitada

CÁLCULO II | UNIDADE 4
pela elipse

Solução: Pelo teorema de Green, temos que

Então, consideremos que o campo e uma parametrização para a elipse


é Assim,

de onde resulta que:

WWW.UNINGA.BR 90
ENSINO A DISTÂNCIA

4 - SUPERFÍCIES
Nas seções anteriores, o conceito de curva lisa foi essencial para a definição da integral
de linha e a formulação do Teorema de Green. Agora, para apresentação dos Teoremas de
Gauss e Stokes, é necessário introduzir o conceito de superfície parametrizada. Uma superfície
parametrizada é uma função

definida e continuamente diferenciável em um domínio de Observe que as


funções e são funções reais continuamente diferenciáveis
em , denominadas as funções coordenadas. O conjunto imagem da superfície parametrizada
( ), que aqui denotamos por é um
subconjunto do espaço chamado o traço da superfície parametrizada . É comum afirmar
que o conjunto está parametrizado por e denotar a superfície por:

CÁLCULO II | UNIDADE 4
Exercício 11: Descreva uma parametrização para a superfície esférica de raio

Solução: Usando coordenadas esféricas temos que a função definida por:

é uma superfície parametrizada, cujo traço é a esfera de centro na origem e raio , pois
. Note que fixado um valor as curvas descrevem os paralelos,
enquanto que fixado um valor as curvas descrevem os semi-meridianos.

WWW.UNINGA.BR 91
ENSINO A DISTÂNCIA

Figura 8 - Ilustração da parametrização da esfera. Fonte: o autor.

Exercício 12: Estabeleça uma parametrização para o Toro, superfície obtido pela rotação
entorno do eixo da circunferência de raio centrada no ponto com

Solução: Nas figuras esboçamos, geometricamente, a construção do Toro:

CÁLCULO II | UNIDADE 4
Figura 9 - Ilustração da parametrização do Toro. Fonte: o autor.

Seja um ponto genérico sobre o Toro. Conforme descrito na figura, o número


é o raio da seção transversal, e o ângulo determina a rotação de nesta seção. Portanto,
vemos que . Projetando este ponto sobre o plano vemos que a distância desta
projeção até a origem é Para encontrarmos a coordenadas e desta projeção,
devemos multiplicar a projeção, respectivamente, por e onde Logo, a
função definida por:

WWW.UNINGA.BR 92
ENSINO A DISTÂNCIA

é uma superfície parametrizada, cujo traço é o Toro.


Considere uma superfície parametrizada

cujo traço denotamos por e sejam e tais


que As retas no plano, paralelas aos eixos coordenados, passando pelo
ponto são parametrizadas por e A restrição de sobre cada
uma dessas retas determina duas curvas sobre parametrizadas por e cujos
vetores tangentes em são dados pelas derivadas:

CÁLCULO II | UNIDADE 4
Figura 10 - Vetores tangentes. Fonte: Google Images (2018).

Observamos que se os vetores e são linearmente independentes (LI), então,


eles determinam um plano. A translação deste plano para o ponto é denominado
plano tangente à superfície parametrizada no ponto (ou às vezes por abuso de linguagem
dizemos plano tangente à no ponto ). Da geometria analítica, sabemos que dois vetores
e são linearmente independentes (LI) se, e somente se, o produto vetorial destes
vetores é não-nulo. Concluímos, assim, que uma condição necessária e suficiente para que a
superfície parametrizada possua plano tangente no ponto é que
Todo ponto nas condições descritas anteriormente é dito um ponto regular de
Nas seções anteriores, quando tratamos sobre curvas definimos curvas lisas como sendo
aquelas curvas que admitem vetor tangente (reta tangente) em todo ponto, estas são diferentes
das extremidades.

WWW.UNINGA.BR 93
ENSINO A DISTÂNCIA

Agora, por analogia, afirmamos que uma superfície parametrizada é uma superfície lisa
(ou regular) se em todo ponto está definido o plano o tangente, ou seja, todo ponto é
ponto regular de Equivalentemente, uma superfície parametrizada é uma superfície lisa (ou
regular), se em todo ponto tem-se que Dados uma superfície lisa
e um ponto o vetor normal unitário à apontando para o exterior, no ponto é o vetor:

CÁLCULO II | UNIDADE 4
Figura 11 - Ilustração de uma superfície lisa. Fonte: Google Images (2018).

Exercício 13: Encontre o vetor normal unitário à esfera

apontando para o exterior, num ponto genérico

Solução: Denotando genericamente temos:

WWW.UNINGA.BR 94
ENSINO A DISTÂNCIA

Portanto, computando o produto vetorial dos vetores acima encontramos:

desde que note que o vetor que é um múltiplo do vetor


posição tem o mesmo sentido deste. Ou seja, aponta para o exterior da
esfera. Também temos que Logo, o vetor normal unitário,
apontando para o exterior, é dado por:

uma superfície lisa, cujo traço denotamos por Adicionalmente, suponha que é
injetiva, exceto, possivelmente, em um número finito de curvas suaves em Então, a área da

CÁLCULO II | UNIDADE 4
superfície é número real obtido pela integral dupla:

Exercício 14: Calcule a área da esfera

Solução: No exercício anterior, vimos que Então:

WWW.UNINGA.BR 95
ENSINO A DISTÂNCIA

Exercício 15: Calcule a área da superfície dada pelo gráfico de uma função ,
definida sobre uma região do plano

Solução: Neste caso, a parametrização usual é a parametrização pelo gráfico dada por
Assim, denotando um ponto genérico
temos:

CÁLCULO II | UNIDADE 4
5 - INTEGRAIS DE SUPERFÍCIE
Iniciamos considerando a integral de superfície de um campo escalar definido sobre o
traço de uma superfície lisa. Mais precisamente, sejam uma região limitada, com área,
e

uma superfície lisa, cujo traço denotamos por Dada uma função suficientemente
regular definimos a integral (de superfície) da função sobre a superfície ,

denotada por , pela fórmula:

Observe que o caso particular em que é a função constante igual a 1, a expressão acima
é a área da superfície Além disso, se representa a densidade superficial de massa, então a

integral é a massa da superfície

WWW.UNINGA.BR 96
ENSINO A DISTÂNCIA

Exercício 16: Calcule a integral onde é a superfície parametrizada por

cujo traço é o cilindro circular reto de raio e altura 1, excluindo a tampa e a base.

Solução: Fazemos e Um simples cálculo nos dá que

Portanto,

Assim,

CÁLCULO II | UNIDADE 4
Agora, consideraremos a integral de superfície de um campo vetorial definido sobre o
traço de uma superfície lisa. Mais precisamente, sejam uma região limitada, com área,
e

uma superfície lisa, cujo traço denotamos por Suponhamos, ainda, que esteja contido
em uma região aberta de onde está definido um campo de vetores
suficientemente regular. Desta forma, a projeção do campo vetorial na direção normal (normal
exterior) da superfície é o campo escalar:

WWW.UNINGA.BR 97
ENSINO A DISTÂNCIA

onde o ponto indica o produto escalar dos vetores e A integral de superfície do


campo escalar sobre a superfície , dada por:

que é comumente chamado de fluxo de através de ou ainda, a integral (de superfície)


da função vetorial sobre a superfície

Exercício 17: Calcule a integral do campo de vetores sobre a


parte do paraboloide , compreendida entre os planos e

CÁLCULO II | UNIDADE 4
Solução: Uma parametrização natural é a parametrização dada pelo gráfico. Seja o
disco de centro na origem e raio 1, isto é, e consideremos
dada por Então, é uma superfície lisa, cujo traço é o
paraboloide entre os planos e Para encontrarmos o campo de vetores normais em
calculamos:

Concluímos, que para esta parametrização o vetor normal:

aponta para o interior do paraboloide. Então, para que tenhamos o campo de vetores
normais unitários, exteriores (apontando para o exterior do paraboloide), devemos multiplicar o
vetor encontrado por -1. Portanto

Assim, a integral desejada é calculada por:

WWW.UNINGA.BR 98
ENSINO A DISTÂNCIA

Para calcular esta última integral, fazemos a mudança para coordenadas polares e
obtemos:

CÁLCULO II | UNIDADE 4
Exercício 18: Encontre o fluxo do campo vetorial através da esfera
.

Solução: Conforme cálculos realizados na resolução do exercício 13, temos que a


parametrização (orientada com vetor normal exterior) da esfera é dada por

de onde,

Vejamos, também, que e, portanto,

Então, o fluxo de F através da esfera é

WWW.UNINGA.BR 99
ENSINO A DISTÂNCIA

6 - TEOREMA DA DIVERGÊNCIA E TEOREMA DE


STOKES

Sejam um subconjunto aberto e

uma função vetorial (campo de vetores) suficientemente regular. Definimos:

i) a divergência do campo F como sendo a função escalar

ii) o rotacional do campo F como sendo a função vetorial

CÁLCULO II | UNIDADE 4
Acima, tanto o quanto as derivadas parciais são calculadas em um ponto genérico
. Omitimos este ponto, na fórmula acima, para simplificar a escrita.

TEOREMA (da Divergência): Seja a região interior de uma superfície fechada,


lisa por partes (suave exceto possivelmente por um número finito de curvas suaves), para qual
podemos associar um campo de vetores normais unitários exteriores (apontando para fora da
superfície – saindo de ). Suponha que seja um
campo de vetores de classe em um conjunto aberto contendo e Então:

isto é, o fluxo do campo através da superfície é a integral (tripla) da divergência de


sobre o sólido

WWW.UNINGA.BR 100
ENSINO A DISTÂNCIA

Observação: É comum denominar o fluxo de um campo vetorial através de uma superfície,


com orientação para fora, por fluxo de saída. Portanto, em palavras, o Teorema da Divergência
afirma que: “O fluxo de saída de um campo vetorial através de uma superfície fechada é igual a
integral tripla da divergência na região envolvida pela superfície”.

Teorema da Divergência, ou Teorema de Gauss


https://www.youtube.com/watch?v=SK9EWk9IxIw

Exercício 19: Calcule o fluxo do campo vetorial através do cubo


unitário.

Solução: Se não usarmos o Teorema da Divergência, o fluxo desejado é obtido por

CÁLCULO II | UNIDADE 4
onde devem ser a parametrização para cada uma das seis arestas do
cubo. Por outro lado, usando o Teorema da Divergência o cálculo fica bem mais simples, ou seja,

Exercício 20: Use o Teorema da Divergência para calcular o fluxo de saída do campo
vetorial através da esfera de centro na origem e raio

Solução: No exercício 18, calculamos o fluxo desejado efetuando diretamente a integral


de superfície. Agora, aplicando o Teorema da Divergência, temos:

onde na última igualdade usamos o valor do volume da bola de raio

Para finalizar nosso estudo, apresentamos uma generalização do Teorema de Green,


estudado na seção 4, para o caso de superfícies em ambiente tridimensional. Está generalização
recebe o nome de Teorema de Stokes.

WWW.UNINGA.BR 101
ENSINO A DISTÂNCIA

Lembramos que o Teorema de Green relaciona o cálculo da integral de linha de um campo


vetorial bidimensional, ao longo de uma curva fechada simples , com uma integral dupla sobre
a região delimitada pela curva O Teorema de Stokes relaciona o cálculo da integral de linha
de um campo vetorial tridimensional, ao longo de uma curva fechada simples , bordo de uma
superfície , com uma integral de superfície sobre

Teorema (de Stokes): Seja uma superfície suave em , limitada por uma curva
fechada Assuma que a superfície está orientada pela normal exterior e a curva o bordo
de então, está orientada de modo que a superfície permaneça à esquerda da curva. Suponha
que é um campo de vetores de classe em um
conjunto aberto contendo a superfície e seu bordo. Então:

CÁLCULO II | UNIDADE 4
Figura 12 - Ilustração da superfície . Fonte: Google Images (2018).

Observação: Quando a superfície é constituída por um número finito de partes


suaves e o bordo também é formado por um número finito de curvas suaves
, então, vale um enunciado análogo, onde é considerada a soma sobre cada uma
dessas partes, isto é:

Observação: O Teorema de Stokes fornece uma interpretação física para o rotacional de


um campo vetorial. Sabemos que a integral de linha que aparece no primeiro membro da fórmula
acima estabelecida no Teorema de Stokes é o trabalho realizado pelo campo vetorial em uma
partícula que percorre a curva Portanto, o Teorema de Stokes afirma que o trabalho realizado
por um campo de forças que percorre, no sentido positivo, uma curva lisa por partes, fechada
e simples pode ser obtido integrando a componente normal do rotacional em uma superfície
orientada delimitada por .

WWW.UNINGA.BR 102
ENSINO A DISTÂNCIA

Exercício 21: Verifique o Teorema de Stokes para o campo vetorial

e a superfície a porção do paraboloide com como na figura.

CÁLCULO II | UNIDADE 4
Figura 13 - Paraboloide com . Fonte: Google Images (2018).

Solução: O bordo de é o circulo no plano , de centro na origem e raio 2, que pode ser
parametrizado por Então:

Assim,

Por outro lado, vejamos o cálculo da integral de superfície. A superfície é o gráfico da


função . Então, conforme fizemos na resolução do exercício 15, uma
parametrização natural para é:

WWW.UNINGA.BR 103
ENSINO A DISTÂNCIA

O rotacional do campo é dado por:

Entao:

Desta forma:

Que mostra a igualdade:

CÁLCULO II | UNIDADE 4

WWW.UNINGA.BR 104
ENSINO A DISTÂNCIA

7 - CONSIDERAÇÕES FINAIS
Vimos que o cálculo vetorial estende o cálculo usual, em espaços euclidianos, para funções
definidas sobre curvas e superfícies. As integrais de linha e de superfície são as generalizações
essenciais. Para curvas lisas (suaves), surgem o conceito da integral de linha de um campo vetorial
sobre a curva e um resultado que estende o Teorema Fundamental do Cálculo, conhecido na
literatura como Teorema de Green. Este importante teorema relaciona uma integral de linha
sobre uma curva lisa por partes, fechada e simples com uma integral dupla sobre a região
delimitada pela curva. Em algumas situações, a aplicação do Teorema de Green simplifica o
cálculo de algumas integrais de linha. Também aparecem aplicações envolvendo o conceito de
trabalho realizado por uma força.
Por outro lado, para superfícies, o interesse é o conceito de fluxo e os resultados principais
são os Teoremas da Divergência (Gauss) e o Teorema de Stokes, que afirmam respectivamente: i)
O fluxo de saída de um campo vetorial através de uma superfície fechada é igual a integral tripla
da divergência na região envolvida pela superfície; ii) O trabalho realizado por um campo de
forças que percorre no sentido positivo uma curva lisa por partes, fechada e simples pode ser
obtido integrando a componente normal do rotacional em uma superfície orientada delimitada
por .

CÁLCULO II | UNIDADE 4

WWW.UNINGA.BR 105
ENSINO A DISTÂNCIA

REFERÊNCIAS
ANTON. H.; BIVENS. I.; DAVIS. S. Cálculo. Vol. 2, 10ª ed. Porto Alegre: Bookman, 2014.

LEITHOLD. L. O Cálculo com Geometria Analítica. Vol. 2. São Paulo: Harbra, 1977.

STEWART. J. Cálculo. Vol. 2, 7a. Ed. São Paulo: Editora Cengage Learning, 2013.

THOMAS. G. B; ROSS. L. F.; MAURICE. D. W.; FRANK. R. G. Cálculo. Vol. 2, 10a. ed. São Paulo:
Editora Pearson, 2003.

WILIAMSON. R. E.; CROWELL. R. H.; TROTTER. H. F. Cálculo de funções vetoriais. Vol. 2.


Livros Técnicos e Científicos. Rio de Janeiro: S. A., 1975.

ZILL. D. G. Equações diferenciais com aplicações em Modelagem. 3a Ed. São Paulo: Cengage
Learning, 2016.

ZILL. D. G.; CULLEN. M. R. Equações diferenciais. Vol. 1, 3a Ed. São Paulo: Pearson Makron
Books, 2006.

WWW.UNINGA.BR 106

Você também pode gostar